You are on page 1of 55

INSTA STATIC QUIZ

SEPTEMBER 2022

WWW.INSIGHTSONINDIA.COM INSIGHTSIAS
INSTA STATIC QUIZ
Table of Contents

1. POLITY ...............................................................................................................................................2

2. GOVERNANCE .................................................................................................................................. 21

3. GEOGRAPHY .................................................................................................................................... 27

4. ECONOMY ....................................................................................................................................... 33

5. ART AND CULTURE ........................................................................................................................... 38

6. HISTORY........................................................................................................................................... 42

7. ENVIRONMENT ................................................................................................................................ 48

www.insightsonindia.com 1
INSTA STATIC QUIZ

1. Polity
1) Consider the following statements.
1. Each village, demarcated as per the Census, must have its own dedicated Gram Sabha and Gram
Panchayat.
2. Anyone who is eligible to vote in Lok Sabha elections from a particular village can also vote in Gram
Panchayat elections.
Which of the above statements is/are correct?
a) 1 only
b) 2 only
c) Both 1 and 2
d) Neither 1 nor 2

Solution: b)

The Gram Sabha is a meeting of all adults who live in the area covered by a Panchayat. This could be only one
village or a few villages. In some states, a village meeting is held for each village.
Anyone who is 18 years old or more and who has the right to vote is a member of the Gram Sabha. These adults
also have a right to vote in Lok Sabha (LS) elections.

2) What was/were the essential changes brought by the 73rd constitutional amendment Act in the Panchayati
Raj system in India?
1. Panchayats were created by Act as they did not exist in Independent India.
2. Panchayati Raj System received constitutional status.
3. It fixed the size of Panchayats and granted executive power to the panchayats.
Select the correct answer code:
a) 1, 2
b) 2 only
c) 1, 3
d) 1, 2, 3

Solution: b)

Rajasthan was the first state to establish Panchayati Raj in 1959. Other states also followed suit.

Though most of the states created panchayati raj institutions by mid 1960s, there were differences from one state
to another with regard to the number of tiers, relative position of samiti and parishad, their tenure, composition,
functions, finances and so on.
The 73rd Act harmonized all the systems into a common structure and granted PRIs constitutional status.

Panchayat is elected from area wards, and its size is not fixed by the constitution.

3) A State Legislature may provide for the representation of which of the following persons in a municipality?
1. members of the Lok Sabha and the state legislative assembly representing constituencies that comprise
wholly or partly the municipal area.
2. Teachers and graduates living within the municipal area
3. Persons having special knowledge or experience in municipal administration
Select the correct answer code:
a) 1, 3
b) 2, 3
c) 1 only
d) 1, 2, 3

Solution: a)
www.insightsonindia.com 2
INSTA STATIC QUIZ

The provision is made to augment municipal administration and the municipality may include:
• Persons having special knowledge or experience in municipal administration without the right to vote in
the meetings of municipality.
• The members of the Lok Sabha and the state legislative assembly representing constituencies that
comprise wholly or partly the municipal area.
• The members of the Rajya Sabha and the state legislative council registered as electors within the
municipal area.
• The chairpersons of committees (other than wards committees).

4) With reference to the ‘Cantonment’ boards, which of the following statements is/are correct?
1. It is established for municipal administration for civilian population in the cantonment area.
2. It works under the administrative control of the Ministry of Housing and Urban Affairs.
Select the correct answer code:
a) 1 only
b) 2 only
c) Both 1 and 2
d) Neither 1 nor 2

Solution: a)

A cantonment board is established for municipal administration for civilian population in the cantonment area.
It is set up under the provisions of the Cantonments Act of 2006—a legislation enacted by the Central
government. It works under the administrative control of the defence ministry of the Central government.

5) The 97th constitutional amendment act


1. Gave a constitutional status and protection to cooperative societies
2. Made the right to form co-operative societies a fundamental right under Article 19
3. Included a new Directive Principle of State Policy on promotion of co-operative societies
Select the correct answer code:
a) 1, 2, 3
b) 1, 3
c) 1, 2
d) 2, 3

Solution: a)

The 97th Constitutional Amendment Act of 2011 gave a constitutional status and protection to co-operative
societies. In this context, it made the following three changes in the constitution:
1. It made the right to form co-operative societies a fundamental right (Article 19).
2. It included a new Directive Principle of State Policy on promotion of cooperative societies (Article 43-B2).
3. It added a new Part IX-B in the Constitution which is entitled “The Cooperative Societies” (Articles 243-ZH to
243-ZT).

6) The NOTA voting option is applicable in


1. State Legislative Assembly Elections
2. Lok Sabha elections
3. Election of the President and Vice-President
Select the correct answer code:
a) 1, 2
b) 1, 3
c) 1, 2, 3
d) 1 only

Solution: a)
www.insightsonindia.com 3
INSTA STATIC QUIZ

"None of the Above" (or NOTA) has been provided as an option to the voters of India in most elections since
2009. The Supreme Court in PUCL vs. Union of India Judgement 2013 directed the use of NOTA in the context of
direct elections to the Lok Sabha and the respective state assemblies.

7) Consider the following statements regarding Election Commission of India.


1. The Constitution has not specified the term of the members of the Election Commission.
2. The Constitution has debarred the retiring election commissioners from any further appointment by
the Government.
3. In case of difference of opinion amongst the Chief Election Commissioner and/or two other election
commissioners, the matter is decided by the Chief Election Commissioner.
Which of the above statements is/are correct?
a) 1 only
b) 1, 2
c) 1, 3
d) 1, 2, 3

Solution: a)

The Constitution has not prescribed the qualifications (legal, educational, administrative or judicial) of the
members of the Election Commission.
The Constitution has not specified the term of the members of the Election Commission.
The Constitution has not debarred the retiring election commissioners from any further appointment by the
government.

The Chief election commissioner and the two other election commissioners have equal powers and receive equal
salary, allowances and other perquisites, which are similar to those of a judge of the Supreme Court.
In case of difference of opinion amongst the Chief Election Commissioner and/or two other election
commissioners, the matter is decided by the Commission by majority.

8) Which of the following are the provisions for independent functioning of the UPSC?
1. The chairman can be removed from office by the president only in the manner and on the grounds
similar to that of a judge of the Supreme Court.
2. The entire expenses of the UPSC are charged on the Consolidated Fund of India.
3. The chairman is not eligible for further employment in the Government of India or a state.
Select the correct answer code:
a) 1, 2
b) 1, 3
c) 2, 3
d) 1, 2, 3

Solution: c)

The Constitution has made the following provisions to safeguard and ensure the independent and impartial
functioning of the UPSC:
(a) The chairman or a member of the UPSC can be removed from office by the president only in the manner and
on the grounds mentioned in the Constitution. Therefore, they enjoy security of tenure.
(b) The conditions of service of the chairman or a member, though determined by the president, cannot be varied
to his disadvantage after his appointment.
(c) The entire expenses including the salaries, allowances and pensions of the chairman and members of the
UPSC are charged on the Consolidated Fund of India. Thus, they are not subject to vote of Parliament.
(d) The chairman of UPSC (on ceasing to hold office) is not eligible for further employment in the Government
of India or a state.
(e) A member of UPSC (on ceasing to hold office) is eligible for appointment as the chairman of UPSC or a State
Public Service Commission (SPSC), but not for any other employment in the Government of India or a state.
www.insightsonindia.com 4
INSTA STATIC QUIZ
(f) The chairman or a member of UPSC is (after having completed his first term) not eligible for reappointment to
that office (i.e., not eligible for second term).

9) Which of these constitutional provisions secure the independence of certain Constitutional bodies from the
government?
1. Security of tenure
2. Fixed service conditions
3. Expenses being charged on the Consolidated Fund of India
Select the correct answer code:
a) 1 only
b) 1, 2, 3
c) 2, 3
d) 1, 2

Solution: b)

The Constitution ensures the independence of these bodies through various provisions like security of tenure,
fixed service conditions, expenses being charged on the Consolidated Fund of India, and so on.
For instance, Chief election Commissioner can only be removed from office on proved misbehaviour or incapacity,
which are the same grounds as that for a Supreme Court Judge.
CEC Service conditions cannot be varied after appointment, and the recommendation of CEC is needed to remove
ECs/regional commissioners.

10) Which of the following bodies have the leader of the opposition in Rajya Sabha, in their appointment
committees?
a) The National Human Rights Commission
b) The Central Vigilance Commission
c) The Central Information Commission
d) CEO of NITI Aayog

Solution: a)

The National Human Rights Commission is a statutory (and not a constitutional) body. It was established in 1993
under a legislation enacted by the Parliament, namely, the Protection of Human Rights Act, 1993.
The chairman and members are appointed by the president on the recommendations of a six-member
committee consisting of the prime minister as its head, the Speaker of the Lok Sabha, the Deputy Chairman of
the Rajya Sabha, leaders of the Opposition in both the Houses of Parliament and the Central home minister.

11) Consider the following statements regarding the Preamble of Indian Constitution.
1. The original Preamble, adopted by the Constituent Assembly in 1949, declared India a “Sovereign
Socialist Secular Democratic Republic”.
2. Preamble states the objects of the Constitution, and acts as an aid during the interpretation of
Articles.in the Constitution.
3. The 42nd Amendment to the Constitution changed “unity of the nation” to “unity and integrity of the
nation”.
Which of the above statements is/are correct?
a) 1, 2
b) 1, 3
c) 2, 3
d) 1, 2, 3

Solution: c)

www.insightsonindia.com 5
INSTA STATIC QUIZ
A preamble is an introductory statement in a document that explains the document’s philosophy and objectives.
In a Constitution, it presents the intention of its framers, the history behind its creation, and the core values and
principles of the nation.

The Preamble is based on the Objective Resolution moved by Jawaharlal Nehru in the Constituent Assembly on
December 13, 1946. The Resolution was adopted on January 22, 1947.

Although not enforceable in court, the Preamble states the objects of the Constitution, and acts as an aid during
the interpretation of Articles when language is found ambiguous.

The original Preamble, adopted by the Constituent Assembly in 1949, declared India a “Sovereign Democratic
Republic”. By the 42nd Amendment of 1976, enacted during the Emergency, the words “Socialist” and “Secular”
were inserted; the Preamble now reads “Sovereign Socialist Secular Democratic Republic”.

The 42nd Amendment to the Constitution, passed in 1976, replaced the words “sovereign democratic republic” to
“sovereign socialist secular democratic republic”. It also changed “unity of the nation” to “unity and integrity of
the nation”.

12) Consider the following statements regarding the Speaker of Lok Sabha.
1. Speaker has to vacate his office if he is removed by a resolution passed by one tenth of all the members
of the Lok Sabha.
2. Whenever the Lok Sabha is dissolved, the Speaker vacates his office and a new speaker is elected when
the newly-elected Lok Sabha meets.
3. He is the guardian of powers and privileges of the Members of the parliament and its committees.
Which of the above statements is/are incorrect?
a) 1 only
b) 1, 2, 3
c) 1, 2
d) 2, 3

Solution: b)

Usually, the Speaker remains in office during the life of the Lok Sabha. However, he has to vacate his office earlier
in any of the following three cases:
1. if he ceases to be a member of the Lok Sabha;
2. if he resigns by writing to the Deputy Speaker; and
3. if he is removed by a resolution passed by a majority of all the members of the Lok Sabha. Such a resolution can
be moved only after giving 14 days’ advance notice.

Whenever the Lok Sabha is dissolved, the Speaker does not vacate his office and continues till the newly-elected
Lok Sabha meets.

The Speaker is the head of the Lok Sabha, and its representative. He is the guardian of powers and privileges of
the members of Lok Sabha, the House as a whole and its committees.

13) Consider the following statements regarding the office of the governor as mentioned in the constitution of
India:
1. His office is constitutionally under the control and subordinate to the Central government.
2. The Governor of a particular state should not belong to that state.
3. The President of India must consult the Chief Minister of the concerned state before making the
appointment of the Governor of that state.
Which of the above statements is/are correct?
a) 1, 2
b) 2, 3
c) 1, 3
www.insightsonindia.com 6
INSTA STATIC QUIZ
d) None of the above

Solution: d)

The governor holds an independent office under the constitution. Options 2 and 3 are conventions and not
mentioned in the constitution. It is solely on the discretion of the President on how he wants to appoint the
Governor (given some qualifications mentioned in the constitution of India)

14) Which one of the following amendments empowered the president to send back the advice of cabinet for
reconsideration?
a) 39th Amendment
b) 40th Amendment
c) 42nd Amendment
d) 44th Amendment

Solution: d)

Forty-Fourth Amendment Act, 1978 empowered the president to send back once the advice of cabinet for
reconsideration. But, the reconsidered advice is to be binding on the president.

15) Consider the following statements about Motion of Thanks.


1. It is addressed by the Leader of the House.
2. The motion is put to vote in both the houses of the parliament.
3. It is addressed at the beginning of every new session of the Parliament.
Which of the above statements is/are incorrect?
a) 1, 2
b) 2, 3
c) 1, 3
d) 1, 2, 3

Solution: c)

Motion of Thanks: The first session after each general election and the first session of every fiscal year is
addressed by the president. In this address, the president outlines the policies and programmes of the
government in the preceding year and ensuing year. This address of the president, is discussed in both the Houses
of Parliament on a motion called the ‘Motion of Thanks’. At the end of the discussion, the motion is put to vote.
This motion must be passed in the House. Otherwise, it amounts to the defeat of the government.

16) Consider the following statements regarding pro- tem speaker.


1. The Constitution of India gives the Governor the power to appoint a pro-tem Speaker in the state
legislature.
2. Powers of the Speaker pro-tem are not co-extensive with the powers of elected Speaker.
Which of the above statements is/are correct?
a) 1 only
b) 2 only
c) Both 1 and 2
d) Neither 1 nor 2

Solution: a)

Article 180 (1) of the Constitution gives the Governor the power to appoint a pro-tem Speaker. The Article says
that if the chair of the Speaker falls vacant and there is no Deputy Speaker to fill the position, the duties of the
office shall be performed “by such member of the Assembly as the Governor may appoint for the purpose”.

www.insightsonindia.com 7
INSTA STATIC QUIZ
The powers of a pro-tem Speaker are wide. The Bombay High Court in its 1994 judgement in the Surendra
Vassant Sirsat case holds that a pro-tem is Speaker of the House “for all purposes with all powers, privileges and
immunities” until the Speaker is elected.
The Odisha High Court also agreed in the Godavaris Misra versus Nandakisore Das, Speaker, Orissa Legislative
Assembly case when it said the “powers of the Speaker pro-tem are co-extensive with the powers of elected
Speaker”.

17) Consider the following statements regarding Fundamental Duties incorporated in the Constitution.
1. All Fundamental Duties were incorporated in Part IV-A of the Constitution by the Constitution 42nd
Amendment Act, 1976.
2. They are not enforceable by law, but a court may take them into account while adjudicating on a
matter.
3. The concept of Fundamental Duties is taken from the Constitution of Russia.
Which of the above statements is/are correct?
a) 1, 2
b) 1, 3
c) 2, 3
d) 1, 2, 3

Solution: c)

The Fundamental Duties were incorporated in Part IV-A of the Constitution by the Constitution 42nd
Amendment Act, 1976. Today, there are 11 Fundamental Duties described under Article 51-A, of which 10 were
introduced by the 42nd Amendment and the 11th was added by the 86th Amendment in 2002.

These are statutory duties, not enforceable by law, but a court may take them into account while adjudicating
on a matter. The idea behind their incorporation was to emphasise the obligation of the citizen in exchange for
the Fundamental Rights that he or she enjoys. The concept of Fundamental Duties is taken from the Constitution
of Russia.

18) Consider the following statements regarding Law Commission of India.


1. The Law Commission of India is a statutory body constituted by the Government of India from time to
time.
2. The commission is re-constituted every five years.
3. The Law Commission shall suo-motu, undertake research in law and review of existing laws in India.
Which of the above statements is/are incorrect?
a) 1, 2
b) 1, 3
c) 2, 3
d) None of the above

Solution: a)

The Law Commission shall, on a reference made to it by the Central Government or suo-motu, undertake
research in law and review of existing laws in India for making reforms therein and enacting new legislations. It
shall also undertake studies and research for bringing reforms in the justice delivery systems for elimination of
delay in procedures, speedy disposal of cases, reduction in cost of litigation etc.

The Law Commission of India is a non-statutory body constituted by the Government of India from time to time.
The Commission was originally constituted in 1955 and is re-constituted every three years.

The various Law Commission have been able to make important contribution towards the progressive
development and codification of Law of the country.
The Law Commission will be constituted for a period of three years from the date of publication of its Order in the
Official Gazette.
www.insightsonindia.com 8
INSTA STATIC QUIZ

19) Consider the following statements regarding the functions of the Law Commission of India.
1. Identify laws which are no longer needed or relevant and can be immediately repealed.
2. Suggest such legislations as might be necessary to implement the Directive Principles and to attain the
objectives set out in the Preamble of the Constitution.
3. Consider the requests for providing research to any foreign countries as may be referred to it by the
Government through Ministry of Law and Justice.
4. Take all such measures as may be necessary to harness law and the legal process in the service of the
poor.
Which of the above statements is/are correct?
a) 1, 2, 3
b) 1, 3, 4
c) 2, 3, 4
d) 1, 2, 3, 4

Solution: d)

The Law Commission of India shall, inter-alia,: -


• identify laws which are no longer needed or relevant and can be immediately repealed;
• examine the existing laws in the light of Directive Principles of State Policy and suggest ways of improvement
and reform and also suggest such legislations as might be necessary to implement the Directive Principles and
to attain the objectives set out in the Preamble of the Constitution;
• consider and convey to the Government its views on any subject relating to law and judicial administration
that may be specifically referred to it by the Government through Ministry of Law and Justice (Department of
Legal Affairs);
• Consider the requests for providing research to any foreign countries as may be referred to it by the
Government through Ministry of Law and Justice (Department of Legal Affairs);
• take all such measures as may be necessary to harness law and the legal process in the service of the poor;
• revise the Central Acts of general importance so as to simplify them and remove anomalies, ambiguities and
inequities;

20) Consider the following statements.


1. A citizen’s right to own private property is a human right.
2. Right to private property ceased to be a fundamental right with the 42nd Constitution Amendment.
Which of the above statements is/are correct?
a) 1 only
b) 2 only
c) Both 1 and 2
d) Neither 1 nor 2

Solution: a)

A citizen’s right to own private property is a human right. The state cannot take possession of it without following
due procedure and authority of law, the Supreme Court has held in a judgment.

The state cannot trespass into the private property of a citizen and then claim ownership of the land in the name
of ‘adverse possession’, the court said.

Property ceased to be a fundamental right with the 44th Constitution Amendment in 1978. Nevertheless, Article
300A required the state to follow due procedure and authority of law to deprive a person of his or her private
property, the Supreme Court said.

21) Which of the following is/are the recommendations of Sarkaria Commission?

www.insightsonindia.com 9
INSTA STATIC QUIZ
1. Article 356 should be used very sparingly, in extreme cases as a last resort when all the available
alternatives fail.
2. The Centre should consult the states before making a law on a subject of the Concurrent List.
3. When the president withholds his assent to the state bills, the reasons should be communicated to the
state government.
Select the correct answer code:
a) 1, 2
b) 2, 3
c) 1, 3
d) 1, 2, 3

Solution: d)

Sarkaria Commission
In 1983, the Central government appointed a three-member Commission on Centre–state relations under the
chairmanship of R S Sarkaria, a retired judge of the Supreme Court.

The Commission made 247 recommendations to improve Centre–state relations. The important
recommendations are mentioned below:
1. A permanent Inter-State Council called the Inter-Governmental Council should be set up under Article 263.
2. Article 356 (President’s Rule) should be used very sparingly, in extreme cases as a last resort when all the
available alternatives fail.
3. When the president withholds his assent to the state bills, the reasons should be communicated to the state
government.
4. The Centre should have powers to deploy its armed forces, even without the consent of states.

22) If any question arises whether a matter falls within the Governor’s discretion or not, whose decision shall be
final and why?
a) Chief Minister because he heads the Council of Ministers
b) State Legislature since it is the highest law-making body within the State
c) Governor of the State because the Constitution confers him this authority
d) President of India who advises Governor of the same

Solution: c)

If any question arises whether a matter falls within the Governor’s discretion or not, decision of the Governor
shall be final, and the validity of anything done by the Governor shall not be called in question on the ground
that he ought or ought not to have acted in his discretion.
Also, the constitution says that the advice tendered by Ministers to the Governor shall not be inquired into in any
court.

23) Consider the following statements regarding Article 32 of the Indian Constitution.
1. Article 32 affirms the right to move the Supreme Court for the enforcement of the rights conferred in
Part III of the Indian Constitution.
2. The right guaranteed under Article 32 is not absolute and can be suspended.
3. An individual approaching the High Court under Article 226 for the violation of fundamental rights is
itself is a fundamental right.
Which of the above statements is/are correct?
a) 1, 2
b) 1, 3
c) 2, 3
d) 3 only

Solution: a)

www.insightsonindia.com 10
INSTA STATIC QUIZ
What is Article 32?
It is one of the fundamental rights listed in the Constitution that each citizen is entitled. Article 32 deals with the
‘Right to Constitutional Remedies’, or affirms the right to move the Supreme Court by appropriate proceedings
for the enforcement of the rights conferred in Part III of the Constitution.

During the 1975 Emergency, a five-judge bench of the Supreme Court, in the ADM Jabalpur vs Shivakant Shukla
case, had ruled that the right to constitutional remedies under Article 32 would remain suspended during a
national emergency.
The 44th Amendment also stated that according to Article 359, the president could issue orders suspending the
right to move any court for the enforcement of fundamental rights, under Article 32, during a national
emergency, with the exception of Article 20 ( deals with protection of certain rights in case of conviction for
offences) and Article 21 (protection of life and personal liberty).

In civil or criminal matters, the first remedy available to an aggrieved person is that of trial courts, followed by an
appeal in the High Court and then the Supreme Court. When it comes to violation of fundamental rights, an
individual can approach the High Court under Article 226 or the Supreme Court directly under Article 32. Article
226, however, is not a fundamental right like Article 32.

24) Consider the following statements.


1. Article 356 is inspired by Government of India Act, 1919.
2. A state government pursuing anti-secular politics is liable to action under Article 356.
3. The presidential proclamation imposing President’s Rule is subject to judicial review.
Which of the above statements is/are correct?
a) 1, 2
b) 1, 3
c) 2, 3
d) 1, 2, 3

Solution: c)

The presidential proclamation imposing President’s Rule is subject to judicial review. The satisfaction of the
President must be based on relevant material. The action of the president can be struck down by the court if it is
based on irrelevant or extraneous grounds or if it was found to be malafide or perverse.

Secularism is one of the ‘basic features’ of the Constitution. Hence, a state government pursuing anti-secular
politics is liable to action under Article 356.

25) Consider the following statements.


1. Right to cast a vote in elections is neither a fundamental right nor a right under common law.
2. Prisoners can cast their vote from jails through postal ballot.
Which of the above statements is/are incorrect?
www.insightsonindia.com 11
INSTA STATIC QUIZ
a) 1 only
b) 2 only
c) Both 1 and 2
d) Neither 1 nor 2

Solution: b)

The Supreme Court had observed that the right to cast a vote in elections is neither a fundamental right
nor a right under common law.

Who can vote and who cannot?


Under Section 62(5) of the Representation of the People Act, 1951, individuals in lawful custody of the police
and those serving a sentence of imprisonment after conviction cannot vote. Undertrial prisoners are also
excluded from participating in elections even if their names are on electoral rolls.
Only those under preventive detention can cast their vote through postal ballots.

26) Consider the following statements regarding the status of Fundamental Rights vs Directive Principles of State
policy.
1. The Supreme Court in Golaknath Case, 1967 held that the Indian Constitution is founded on the
bedrock of the balance between the Fundamental Rights and the Directive Principles.
2. The Parliament can completely amend Directive Principles of State policy in order to improve the
administration of any of the Fundamental Rights.
Which of the above statements is/are correct?
a) 1 only
b) 2 only
c) Both 1 and 2
d) Neither 1 nor 2

Solution: d)

In the Minerva Mills case (1980), the Supreme Court also held that ‘the Indian Constitution is founded on the
bedrock of the balance between the Fundamental Rights and the Directive Principles.

This harmony and balance between the two is an essential feature of the basic structure of the Constitution.

The Parliament can amend the Fundamental Rights for implementing the Directive Principles, so long as the
amendment does not damage or destroy the basic structure of the Constitution.
However, the Parliament cannot amend the Directive principles on its whims and fancies and affect the
‘welfare state’ credential of Indian constitution since it forms a part of the basic structure.

27) Consider the following statements regarding Governor’s legislative powers.


1. If a bill passed by the state legislature endangers the position of the state high court, the Governor
shall reserve the bill for consideration of the President.
2. If a bill sent by Governor for the reconsideration of the State legislature is passed again without
amendments, the Governor is under no constitutional obligation to give his assent to the bill.
Which of the above statements is/are incorrect?
a) 1 only
b) 2 only
c) Both 1 and 2
d) Neither 1 nor 2

Solution: b)

Article 200 provides that when a Bill passed by the State Legislature, is presented to the Governor, the Governor
shall declare—
www.insightsonindia.com 12
INSTA STATIC QUIZ
(a) that he assents to the Bill; or
(b) that he withholds assent therefrom; or
(c) that he reserves the Bill for the President's consideration; or
(d) the Governor may, as soon as possible, return the Bill (other than a Money Bill) with a message for re-
consideration by the State Legislature. But, if the Bill is again passed by the Legislature with or without
amendment, the Governor shall not withhold assent therefrom; or
(e) if in the opinion of the Governor, the Bill, if it became law, would so derogate from the powers of the High
Court as to endanger its constitutional position, he shall not assent to but shall reserve it for the consideration of
the President.

28) Consider the following statements regarding the ‘sealed covers’ submitted by the government or its agencies
to the Supreme Court during case hearing.
1. The practice of submission of confidential material to the court in sealed covers is not backed by any
law.
2. The Government has the privilege of non-disclosure of some documents and communications on the
ground of national security.
3. The action taken report on any case cannot be submitted to the court in a sealed cover.
Which of the above statements is/are correct?
a) 2 only
b) 1, 2
c) 2, 3
d) 1, 3

Solution: a)

In the Muzaffarpur shelter home sexual abuse case, the former Chief Justice N.V. Ramana wondered why even an
‘action taken’ report should be in a sealed envelope.

It is true that the law permits the submission of confidential material to the court in some cases. In addition,
courts can order some contents to be kept confidential. The Evidence Act also allows the privilege of non-
disclosure of some documents and communications.

29) Part XVII of the Constitution deals with the official language. Its provisions are divided into
1. Official Language of the Union
2. Regional languages
3. Language of the judiciary and texts of laws
Select the correct answer code:
a) 1, 2
b) 1 only
b) 1, 3
d) 1, 2, 3

Solution: d)

Part XVII of the Constitution deals with the official language in Articles 343 to 351. Its provisions are divided into
four heads—
• Language of the Union,
• Regional languages,
• Language of the judiciary and texts of laws and
• Special directives.

30) The Constitution of India lays down a functional separation of the organs of the State in which of the
following manner?

www.insightsonindia.com 13
INSTA STATIC QUIZ
1. The President or the Governor shall not be answerable to any court for the exercise and performance
of the powers and duties of his office.
2. The validity of proceedings in Parliament and the Legislatures cannot be called into question in any
Court.
3. Parliament cannot discuss the conduct of the judges except when the proceeding to remove a judge is
being carried out.
Select the correct answer code:
a) 1, 2
b) 2, 3
b) 1, 3
d) 1, 2, 3

Solution: d)

The Constitution of India lays down a functional separation of the organs of the State in the following manner:
• Article 50: State shall take steps to separate the judiciary from the executive. This is for the purpose of
ensuring the independence of judiciary.
• Article 122 and 212: validity of proceedings in Parliament and the Legislatures cannot be called into
question in any Court. This ensures the separation and immunity of the legislatures from judicial
intervention on the allegation of procedural irregularity.
• Judicial conduct of a judge of the Supreme Court and the High Courts’ cannot be discussed in the
Parliament and the State Legislature, according to Article 121 and 211 of the Constitution.
• Articles 53 and 154 respectively, provide that the executive power of the Union and the State shall be
vested with the President and the Governor and they enjoy immunity from civil and criminal liability.
• Article 361: the President or the Governor shall not be answerable to any court for the exercise and
performance of the powers and duties of his office.

31) Which of the following principles is considered as the bedrock principle of parliamentary government?
a) Liberty
b) Sovereignty
c) Fraternity
d) Collective Responsibility

Solution: d)

The principle of collective responsibility implies that the Lok Sabha can remove the ministry (i.e., council of
ministers headed by the prime minister) from office by passing a vote of no confidence. This is the bedrock
principle of parliamentary government. The ministers are collectively responsible to the Parliament in general
and to the Lok Sabha in particular (Article 75). They act as a team, and swim and sink together.

32) Consider the following statements regarding Double Membership in Houses of Parliament
1. If a person is elected to both the Houses of Parliament, he should exercise his option for one,
otherwise, both seats become vacant.
2. If a person is elected to two seats in a House, he should exercise his option for one, otherwise, both
seats become vacant.
3. If a sitting member of one House is also elected to the other House, his seat in the first House becomes
vacant.
Which of the above statements is/are correct?
a) 1, 2
b) 1, 3
c) 2, 3
d) 1, 2, 3

Solution: c)

www.insightsonindia.com 14
INSTA STATIC QUIZ

A person cannot be a member of both Houses of Parliament at the same time. Thus, the Representation of People
Act (1951) provides for the following:
(a) If a person is elected to both the Houses of Parliament, he must intimate within 10 days in which House he
desires to serve. In default of such intimation, his seat in the Rajya Sabha becomes vacant.
(b) If a sitting member of one House is also elected to the other House, his seat in the first House becomes
vacant.
(c) If a person is elected to two seats in a House, he should exercise his option for one. Otherwise, both seats
become vacant.
Similarly, a person cannot be a member of both the Parliament and the state legislature at the same time. If a
person is so elected, his seat in Parliament becomes vacant if he does not resign his seat in the state legislature
within 14 days.

33) Consider the following statements.


1. The laws made by the Parliament are applicable to all the states of India.
2. Under no circumstance, the laws made by the state legislature are applicable outside the state.
3. The laws of the Parliament are applicable to the Indian citizens and their property in any part of the
world.
Which of the above statements is/are correct?
a) 1 only
b) 1, 2
c) 1, 3
d) 1, 2, 3

Solution: c)

The Parliament can make laws for the whole or any part of the territory of India. The territory of India includes
the states, the union territories, and any other area for the time being included in the territory of India.
A state legislature can make laws for the whole or any part of the state. The laws made by a state legislature are
not applicable outside the state, except when there is a sufficient nexus between the state and the object.
The Parliament alone can make ‘extra-territorial legislation’. Thus, the laws of the Parliament are also
applicable to the Indian citizens and their property in any part of the world.

34) The mutual delegation of executive power between Centre and States cannot occur
a) From State to Centre through Governor
b) From Centre to State through President
c) From Centre to State through Parliament
d) From State to Centre through State Legislature

Solution: d)

• The Constitution provides for inter-government delegation of executive functions in order to mitigate rigidity
and avoid a situation of deadlock.
• Accordingly, the President may, with the consent of the state government, entrust to that government any of
the executive functions of the Centre.
• Conversely, the governor of a state may, with the consent of the Central government, entrust to that
government any of the executive functions of the state. This mutual delegation of administrative functions
may be conditional or unconditional.
• The Constitution also makes a provision for the entrustment of the executive functions of the Centre to a
state without the consent of that state. But, in this case, the delegation is by the Parliament and not by the
president.
• Thus, a law made by the Parliament on a subject of the Union List can confer powers and impose duties on a
state, or authorise the conferring of powers and imposition of duties by the Centre upon a state (irrespective
of the consent of the state concerned).
• Notably, the same thing cannot be done by the state legislature.
www.insightsonindia.com 15
INSTA STATIC QUIZ

35) Article 355 of the constitution places which of the following duties on the Central Government?
1. Protect every state against external aggression and internal disturbance
2. Ensuring that every state government follows constitutional provisions
3. Ensuring that every state is protected against economic downturns
Select the correct answer code:
a) 1, 2
b) 1, 3
c) 1 only
d) 1, 2, 3

Solution: a)

Constitution imposes two duties on the Centre: (a) to protect every state against external aggression and internal
disturbance; and (b) to ensure that the government of every state is carried on in accordance with the provisions
of the Constitution.
It is this duty in the performance of which the Centre takes over the government of a state under Article 356 in
case of failure of constitutional machinery in state. This is popularly known as ‘President’s Rule’. It is also known
as ‘State Emergency’ or ‘Constitutional Emergency’.

36) Consider the following statements regarding Objectives Resolution.


1. Jawaharlal Nehru moved the historic ‘Objectives Resolution’ in the Constituent Assembly and was
unanimously adopted by the Assembly in 1950.
2. Its modified version forms the Preamble of the present Constitution.
Which of the above statements is/are correct?
a) 1 only
b) 2 only
c) Both 1 and 2
d) Neither 1 nor 2

Solution: b)

On December 13, 1946, Jawaharlal Nehru moved the historic ‘Objectives Resolution’ in the Assembly. It laid
down the fundamentals and philosophy of the constitutional structure.

This Resolution was unanimously adopted by the Assembly on January 22, 1947.
It influenced the eventual shaping of the constitution through all its subsequent stages. Its modified version
forms the Preamble of the present Constitution.

37) Consider the following statements regarding Major Committees of Constituent Assembly.
1. Union Powers Committee was headed by Jawaharlal Nehru.
2. Provincial Constitution Committee was headed by Dr. B.R. Ambedkar.
3. Advisory Committee on Fundamental Rights was headed by Sardar Vallabhbhai Patel.
Which of the above statements is/are correct?
a) 1, 2
b) 1 only
c) 1, 3
d) 2, 3

Solution: c)

The Constituent Assembly appointed a number of committees to deal with different tasks of constitution-making.
Out of these, eight were major committees and the others were minor committees. The names of these
committees and their Chairman are given below:
1. Union Powers Committee – Jawaharlal Nehru
www.insightsonindia.com 16
INSTA STATIC QUIZ
2. Union Constitution Committee -Jawaharlal Nehru.
3. Provincial Constitution Committee -Sardar Patel
4. Drafting Committee – Dr. B.R. Ambedkar
5. Advisory Committee on Fundamental Rights, Minorities and Tribal and Excluded Areas – Sardar Patel.
6. Rules of Procedure Committee – Dr. Rajendra Prasad
7. States Committee (Committee for Negotiating with States) – Jawaharlal Nehru
8. Steering Committee – Dr. Rajendra Prasad

38) Which of the following provisions in the Indian constitution describes India as Secular State?
1. No person shall be compelled to pay any taxes for the promotion of a particular religion.
2. The State shall not deny to any person equality before the law or equal protection of the laws.
3. Equality of opportunity for all citizens in matters of public employment.
Select the correct answer code:
a) 1, 2
b) 1, 3
c) 2, 3
d) 1, 2, 3

Solution: d)

The Constitution of India stands for a Secular State. Hence, it does not uphold any particular religion as the official
religion of the Indian State. The following provisions of the Constitution reveal the secular character of the Indian
State:
(a) The term ‘secular’ was added to the Preamble of the Indian Constitution by the 42nd Constitutional
Amendment Act of 1976.
(b) The Preamble secures to all citizens of India liberty of belief, faith and worship.
(c) The State shall not deny to any person equality before the law or equal protection of the laws (Article 14).
(d) The State shall not discriminate against any citizen on the ground of religion (Article 15).
(e) Equality of opportunity for all citizens in matters of public employment (Article 16).
(f) All persons are equally entitled to freedom of conscience and the right to freely profess, practice and
propagate any religion (Article 25).
(g) Every religious denomination or any of its section shall have the right to manage its religious affairs (Article
26).
(h) No person shall be compelled to pay any taxes for the promotion of a particular religion (Article 27).
(i) No religious instruction shall be provided in any educational institution maintained by the State (Article 28).
(j) Any section of the citizens shall have the right to conserve its distinct language, script or culture (Article 29).
(k) All minorities shall have the right to establish and administer educational institutions of their choice (Article
30).
(l) The State shall endeavour to secure for all the citizens a Uniform Civil Code (Article 44).

39) When the President’s Rule is imposed in a state


1. the President dismisses the state council of ministers headed by the chief minister.
2. the President only suspends and not dissolve the state legislative assembly.
3. the Parliament passes the state legislative bills and the state budget.
Select the correct answer code:
a) 1 only
b) 1, 2
c) 1, 3
d) 1, 2, 3

Solution: c)

When the President’s Rule is imposed in a state, the President dismisses the state council of ministers headed
by the chief minister.

www.insightsonindia.com 17
INSTA STATIC QUIZ
The state governor, on behalf of the President, carries on the state administration with the help of the chief
secretary of the state or the advisors appointed by the President.
This is the reason why a proclamation under Article 356 is popularly known as the imposition of ‘President’s
Rule’ in a state. Further, the President either suspends or dissolves the state legislative assembly.
The Parliament then passes the state legislative bills and the state budget.

40) Consider the following statements regarding Deputy Chairman of Rajya Sabha.
1. He acts as the Chairman only when the Vice president is absent from the sitting of the House.
2. The Deputy Chairman is subordinate to the Chairman of Rajya Sabha.
3. Like the Chairman, the Deputy Chairman, while presiding over the House, cannot vote in the first
instance.
Which of the above statements is/are correct?
a) 1, 2
b) 1, 3
c) 2, 3
d) 3 only

Solution: d)

The Deputy Chairman is elected by the Rajya Sabha itself from amongst its members. Whenever the office of the
Deputy Chairman falls vacant, the Rajya Sabha elects another member to fill the vacancy.

The Deputy Chairman vacates his office in any of the following three cases:
• if he ceases to be a member of the Rajya Sabha;
• if he resigns by writing to the Chairman; and
• if he is removed by a resolution passed by a majority of all the then members of the Rajya Sabha. Such a
resolution can be moved only after giving 14 days’ advance notice.

The Deputy Chairman performs the duties of the Chairman’s office when it is vacant or when the Vice-President
acts as President or discharges the functions of the President. He also acts as the Chairman when the latter is
absent from the sitting of the House.
In both the cases, he has all the powers of the Chairman. It should be emphasised here that the Deputy
Chairman is not subordinate to the Chairman.
He is directly responsible to the Rajya Sabha.

Like the Chairman, the Deputy Chairman, while presiding over the House, cannot vote in the first instance; he
can only exercise a casting vote in the case of a tie. Further, when a resolution for the removal of the Deputy
Chairman is under consideration of the House, he cannot preside over a sitting of the House, though he may be
present.

41) Consider the following statements regarding Election Commission of India.


1. The Constitution has not prescribed the qualifications of the members of the Election Commission.
2. The Constitution has specified the term of the members of the Election Commission.
3. The Constitution has debarred the retiring election commissioners from any further appointment by
the government.
Which of the above statements is/are correct?
a) 1, 2
b) 1, 3
c) 1 only
d) 2, 3

Solution: c)

Though the constitution has sought to safeguard and ensure the independence and impartiality of the Election
Commission, some flaws with respect to the members of the EC can be noted, viz.,
www.insightsonindia.com 18
INSTA STATIC QUIZ
• The Constitution has not prescribed the qualifications (legal, educational, administrative or judicial) of
the members of the Election Commission.
• The Constitution has not specified the term of the members of the Election Commission.
• The Constitution has not debarred the retiring election commissioners from any further appointment
by the government.

42) Consider the following statements.


1. Article 324 of the Constitution provides the same protection to all the election commissioners as the
chief election commissioner.
2. The chief election commissioner can only be removed on the same grounds as a judge of the Supreme
Court.
Which of the above statements is/are correct?
a) 1 only
b) 2 only
c) Both 1 and 2
d) Neither 1 nor 2

Solution: b)

The CEC and the Election Commissioners have a tenure of six years, or up to the age of 65, whichever is earlier,
and enjoy the same status and receive salary and perks as available to Supreme Court judges.
The CEC and the Election Commissioners enjoy the same decision-making powers which is suggestive of the fact
that their powers are at par with each other.
However, Article 324(5) does not provide similar protection to the Election Commissioners and it merely says
that they cannot be removed from office except on the recommendation of the CEC.

43) Consider the following statements regarding Finance Commission of India.


1. Finance Commission is set up under Article 280 of the Constitution.
2. Union Finance Minister acts as the ex officio member of the commission.
3. The commission addresses the imbalances that arise between the taxation powers and expenditure
responsibilities of the centre and the states, respectively.
Which of the above statements is/are correct?
a) 1 only
b) 1, 2
c) 2, 3
d) 1, 3

Solution: d)

• FC is a body set up under Article 280 of the Constitution.


• Its primary job is to recommend measures and methods on how revenues need to be distributed between the
Centre and states.
• There are no-ex officio members of the commission as such, because it is constituted afresh every five years.
• Besides suggesting the mechanism to share tax revenues, the Commission also lays down the principles for
giving out grant-in-aid to states and other local bodies.
• The commission has to take on itself the job of addressing the imbalances that often arise between the
taxation powers and expenditure responsibilities of the centre and the states, respectively.

44) Consider the following statements regarding Food Corporation of India.


1. It is a statutory body under the Ministry of Agriculture and Farmers Welfare.
2. It has primary duty to undertake purchase, store, transport, distribute and sell food grains and other
foodstuffs.
Which of the above statements is/are incorrect?
a) 1 only

www.insightsonindia.com 19
INSTA STATIC QUIZ
b) 2 only
c) Both 1 and 2
d) Neither 1 nor 2

Solution: a)

The Food Corporation of India is an organization created and run by the Government of India. It is a statutory
body under the Ministry of Consumer Affairs, Food and Public Distribution, Government of India.
It has primary duty to undertake purchase, store, and move/transport, distribute and sell food grains and other
foodstuffs.

45) A bill is not deemed to be a money bill if it provides for:


1. Imposition of fines or other pecuniary penalties
2. Receipt of money on account of the public account of India.
3. Imposition, abolition, remission, alteration or regulation of any tax by any local authority or body for
local purposes.
Select the correct answer code:
a) 1 only
b) 1, 2
c) 1, 3
d) 2, 3

Solution: c)

Article 110 of the Constitution deals with the definition of money bills. It states that a bill is deemed to be a
money bill if it contains ‘only’ provisions dealing with all or any of the following matters:
• The imposition, abolition, remission, alteration or regulation of any tax;
• The regulation of the borrowing of money by the Union government;
• The custody of the Consolidated Fund of India or the contingency fund of India, the payment of moneys
into or the withdrawal of money from any such fund;
• The appropriation of money out of the Consolidated Fund of India;
• Declaration of any expenditure charged on the Consolidated Fund of India or increasing the amount of
any such expenditure;
• The receipt of money on account of the Consolidated Fund of India or the public account of India or the
custody or issue of such money, or the audit of the accounts of the Union or of a state; or
• Any matter incidental to any of the matters specified above.

However, a bill is not to be deemed to be a money bill by reason only that it provides for:
• the imposition of fines or other pecuniary penalties, or
• the demand or payment of fees for licenses or fees for services rendered; or
• the imposition, abolition, remission, alteration or regulation of any tax by any local authority or body
for local purposes.

www.insightsonindia.com 20
INSTA STATIC QUIZ

2. Governance
1) Which of the following could be the reasons for the failure of infrastructure projects by the public sector
enterprises?
1. Ineffective Governance
2. Cost overruns.
3. Optimum input-output ratio is rarely observed leading to overcapitalisation
4. Lack of inter-ministerial/departmental coordination
Select the correct answer code:
a) 1, 2, 3
b) 1, 3, 4
c) 2, 3, 4
d) 1, 2, 3, 4

Solution: d)

The primary reasons for the failure of public sector enterprises is no secret. Cost overruns, inter alia, is one of the
major reasons. In some cases, project completion time is exceeded, leading to elevated project cost so much so
that either the project itself becomes unviable at the time of its launching or delays its break even point. Besides,
optimum input-output ratio is seldom observed in a majority of government infrastructural projects leading to
their overcapitalisation. A reluctance to implement labour reforms, a lack of inter-ministerial/departmental
coordination, poor decision-making, ineffective governance and excessive government control are other reasons
for the failure of public infrastructural assets.

2) Which of the following employees are covered by the Central Administrative Tribunal (CAT)?
a) Secretarial staff of the Parliament
b) Officers and servants of the Supreme Court
c) Members of the defence forces
d) None of the above

Solution: d)

The Central Administrative Tribunal (CAT) was set up in 1985 with the principal bench at Delhi and additional
benches in different states.
The CAT exercises original jurisdiction in relation to recruitment and all service matters of public servants
covered by it. Its jurisdiction extends to the all-India services, the Central civil services, civil posts under the
Centre and civilian employees of defence services. However, the members of the defence forces, officers and
servants of the Supreme Court and the secretarial staff of the Parliament are not covered by it.

3) Which of the following are the benefits of Citizens’ Charter?


1. Creates Accountability on Individuals and Organisations providing services.
2. Value for the taxpayers’ money
3. Improves the quality of services
4. Specify what to expect and how to act if standards are not met
Select the correct answer code:
a) 1, 2, 3
b) 1, 3, 4
c) 1, 2, 3, 4
d) 2, 3, 4

Solution: c)

The basic principles of citizens’ charter movement and its importance

www.insightsonindia.com 21
INSTA STATIC QUIZ
It was first articulated and implemented in the UK in 1991 as a national programme. The basic objective of the
Citizens’ Charter is to empower the citizen in relation to public service delivery. Six principles of the Citizens’
Charter movement as originally framed, were: (i) Quality: Improving the quality of services; (ii) Choice: Wherever
possible; (iii) Standards: Specify what to expect and how to act if standards are not met; (iv) Value: For the
taxpayers’ money; (v) Accountability: Individuals and Organisations; and (vi) Transparency: Rules/ Procedures/
Schemes/Grievances.

“Citizens’ Charters” initiative is a response to the quest for solving the problems which a citizen encounters while
dealing with the organisations providing public services. The concept of Citizens’ Charter enshrines the trust
between the service provider and its users.

4) The initiative DARPAN, sometimes seen in news is related to


a) Improve the quality and governance of school education
b) Platform for all voluntary organisations/NGOs engaged in development activities.
c) Expand immunization coverage across the country.
d) Disaster preparedness, and responses during health emergencies

Solution: b)

NGO-DARPAN is a platform that provides space for interface between Non-Government organizations (NGOs)/
Voluntary Organizations (VOs) in the country and key Government Ministries / Departments / Government
Bodies.

It started out as an initiative of the Prime Minister's Office, to create and promote a healthy partnership between
NGOs/VOs and the Government of India. Now it is an e-governance application offered by NITI Aayog to
electronically maintain data and transparency regarding NGOs/VOs in the country.

5) Consider the following statements regarding e-Courts Project.


1. e-Courts Project is an Integrated Mission Mode Project, part of National e-Governance Plan.
2. It is conceptualized on the basis of ‘National Policy and Action Plan for Implementation of Information
and Communication Technology in Indian Judiciary’.
3. It is monitored and funded by the Union Home Ministry.
Which of the above statements is/are correct?
a) 1, 2
b) 1, 3
c) 2, 3
d) 1, 2, 3

Solution: a)

As part of National e-Governance Plan, e-Courts Project is an Integrated Mission Mode Project under
implementation since 2007 for the ICT development of the Indian Judiciary based on the ‘National Policy and
Action Plan for Implementation of Information and Communication Technology in Indian Judiciary’.

It is a Pan-India Project, monitored and funded by Department of Justice, Ministry of Law and Justice,
Government of India for the District Courts across the country.

6) The Social Security Code of 2020 brings which of the following under the ambit of social security schemes,
including for their insurance and health?
1. Unorganised sector
2. Gig workers
3. Platform workers
Select the correct answer code:
a) 1 only
b) 1, 2
www.insightsonindia.com 22
INSTA STATIC QUIZ
c) 1, 3
d) 1, 2, 3

Solution: d)

The Social Security Code of 2020 brings the unorganised sector, gig workers and platform workers under the
ambit of social security schemes, including for their insurance and health.

7) Consider the following statements regarding The Consumer Protection Act, 2019.
1. The Act recognises offences such as providing false information regarding the quality or quantity of a
good or service, and misleading advertisement.
2. Central Consumer Protection Authority is constituted under the Act, which is an advisory body.
3. Central Consumer Protection Authority will be headed by the Ministry of Consumer Affairs, Food and
Public Distribution.
Which of the above statements is/are correct?
a) 1 only
b) 1, 2
c) 2, 3
d) 1, 2, 3

Solution: a)

Central Consumer Protection Authority constituted under Section 10(1) of The Consumer Protection Act, 2019.
The Act replaced The Consumer Protection Act, 1986, and seeks to widen its scope in addressing consumer
concerns. The new Act recognises offences such as providing false information regarding the quality or quantity
of a good or service, and misleading advertisements. It also specifies action to be taken if goods and services are
found “dangerous, hazardous or unsafe”.

The CCPA aims to protect the rights of the consumer by cracking down on unfair trade practices, and false and
misleading advertisements that are detrimental to the interests of the public and consumers.

The CCPA will have the powers to inquire or investigate into matters relating to violations of consumer rights or
unfair trade practices suo motu, or on a complaint received, or on a direction from the central government.

CCPA will have Chief Commissioner as head, and two other commissioners as members.

The CCPA will have an Investigation Wing that will be headed by a Director General.

8) The Department of Consumer Affairs is entrusted with the implementation of


1. Bureau of Indian Standards Act, 2016
2. Standards of Weights and Measures
3. Essential Commodities Act, 1955
4. Emblems and Names (Prevention of Improper Use) Act, 1952
5. Prevention of Black Marketing
Select the correct answer code:
a) 1, 2, 3, 5
b) 2, 3, 5
d) 2, 3, 4
d) 1, 2, 3, 4, 5

Solution: d)

Department of Consumer Affairs is one of the two Departments under the Ministry of Consumer Affairs, Food &
Public Distribution. It was constituted as a separate Department in June 1997 as it was considered necessary to
have a separate Department to give a fillip to the nascent consumer movement in the country.
www.insightsonindia.com 23
INSTA STATIC QUIZ

THE DEPARTMENT HAS BEEN ENTRUSTED WITH THE FOLLOWING WORK


• Implementation of Consumer Protection Act, 2019.
• Implementation of Bureau of Indian Standards Act, 2016
• Implementation of Standards of Weights and Measures - The Legal Metrology Act, 2009.
• Regulation of Packaged Commodities.
• The Essential Commodities Act, 1955 (10 of 1955) (Supply, Prices and Distribution of Essential
Commodities not dealt with specifically by any other Department).
• Prevention of Black Marketing and Maintenance of Supply of Essential Commodities Act, 1980(7 of
1980).
• Monitoring of prices and availability of essential commodities.
• Direct Selling
• Training in Legal Metrology.
• The Emblems and Names (Prevention of Improper Use) Act, 1952.
• Laying down specifications, standards and codes and ensuring quality control of bio-fuels for end uses.
• Consumer Cooperatives
• National Test House.

9) Consider the following statements.


1. In India according to Information Technology Act 2000, an admin of any messaging service say
WhatsApp can be held liable for a post made by a member in the group.
2. Vicarious liability means, the responsibility imposed on one person for the wrongful actions of another
person.
3. Vicarious liability is applicable to only civil cases and not criminal cases.
Which of the above statements is/are correct?
a) 1, 2
b) 2 only
c) 2, 3
d) 1, 2, 3

Solution: b)

In the absence of a special penal law creating vicarious liability, an admin of a WhatsApp group cannot be held
liable for the objectionable post by a group member. There is no law by which an admin of any messaging
service can be held liable for a post made by a member in the group. A WhatsApp admin cannot be an
intermediary under the IT Act.

Vicarious liability in a broader sense, is the responsibility imposed on one person for the wrongful actions of
another person. This often occurs in the context of civil law—for example, in employment cases. In a criminal
context, vicarious liability assigns guilt, or criminal liability, to a person for wrongful acts committed by someone
else.

10) Consider the following statements regarding National Policy for Rare Diseases, 2021.
1. WHO defines rare disease as having a frequency of less than 1 per 10,000 people.
2. Financial support under Rashtriya Arogaya Nidhi shall be provided by the Central Government for
treatment of rare diseases.
3. The policy will make use of a crowdfunding mechanism to cover the cost of treatment of rare diseases.
Which of the above statements is/are correct?
a) 1, 2
b) 2 only
c) 2, 3
d) 1, 2, 3

Solution: c)
www.insightsonindia.com 24
INSTA STATIC QUIZ

WHO defines rare disease as having a frequency of less than 6.5-10 per 10,000 people.

National Policy for Rare Diseases, 2021:


• Financial support under the Umbrella Scheme of Rashtriya Arogaya Nidhi shall be provided by the
Central Government for treatment, of those rare diseases that require a one-time treatment.
Beneficiaries for such financial assistance would not be limited to BPL families, but extended to about
40% of the population, who are eligible as per norms of Pradhan Mantri Jan Arogya Yojana, for their
treatment in Government tertiary hospitals only.
• Government will endeavour to create alternate funding mechanism through setting up a digital platform
for voluntary individual and corporate donors to contribute to the treatment cost of patients of rare
diseases.
• Voluntary crowd-funding for treatment.

11) Consider the following statements.


1. Under Drug (Price Control) Order, National Pharmaceutical Pricing Authority (NPPA) cannot allow
Pharmaceutical companies to raise the prices of essential medicines.
2. The Drugs Prices Control Order is an order issued by the Government of India under Essential
Commodities Act, 1955 to regulate the prices of drugs.
3. Not all the drugs marketed in the country are under price control.
Which of the above statements is/are correct?
a) 1, 3
b) 2 only
c) 2, 3
d) 1, 2, 3

Solution: c)

The National Pharmaceutical Pricing Authority (NPPA) can use its emergency powers to allow companies to
raise the price of drugs.

The Drugs Prices Control Order is an order issued by the Government of India under Sec. 3 of Essential
Commodities Act, 1955 to regulate the prices of drugs.
The Order interalia provides the list of price-controlled drugs, procedures for fixation of prices of drugs, method
of implementation of prices fixed by Govt., penalties for contravention of provisions etc.

Are all the drugs marketed in the country under price control?
No. The National List of Essential Medicines (NLEM) 2011 is adopted as the primary basis for determining
essentiality, which constitutes the list of scheduled medicines for the purpose of price control. The DPCO 2013
contains more than 600 scheduled drug formulations spread across 27 therapeutic groups. However, the prices of
other drugs can be regulated, if warranted in public interest.

12) National Institute Ranking Framework (NIRF) assesses education institutions on which of the following
parameters?
1. Teaching, learning and resources
2. Research and professional practices
3. Graduation outcomes
4. Outreach and inclusivity
Select the correct answer code:
a) 1, 2, 3
b) 1, 2
c) 1, 2, 4
d) 1, 2, 3, 4

Solution: d)
www.insightsonindia.com 25
INSTA STATIC QUIZ

In order to be ranked, all education institutions are assessed on five parameters: teaching, learning and
resources, research and professional practices, graduation outcomes, outreach and inclusivity, and perception.

13) Consider the following statements regarding Genetic Engineering Appraisal Committee (GEAC).
1. The Genetic Engineering Appraisal Committee (GEAC) functions in the Ministry of Science and
Technology.
2. It is responsible for appraisal of activities involving large scale use of hazardous microorganisms and
recombinants in research and industrial production from the environmental angle.
3. It is chaired by a representative from the Department of Biotechnology (DBT).
Which of the above statements is/are correct?
a) 1, 2
b) 2 only
c) 2, 3
d) 1, 2, 3

Solution: b)

The Genetic Engineering Appraisal Committee (GEAC) functions in the Ministry of Environment, Forest and
Climate Change (MoEF&CC). As per Rules, 1989, it is responsible for appraisal of activities involving large scale
use of hazardous microorganisms and recombinants in research and industrial production from the
environmental angle. The committee is also responsible for appraisal of proposals relating to release of
genetically engineered (GE) organisms and products into the enviornment including experimental field trials.
GEAC is chaired by the Special Secretary/Additional Secretary of MoEF&CC and co-chaired by a representative
from the Department of Biotechnology (DBT).

14) Consider the following statements regarding Pradhan Mantri Fasal Bhima Yogna (PMFBY).
1. PMFBY is a central-state scheme which aims to cushion farmers against crop loss.
2. Under the scheme, farmers only bear 1.5 to 5 per cent of the premium.
3. The scheme is made mandatory for farmers who availed institutional finance.
Which of the above statements is/are correct?
a) 1 only
b) 1, 2
c) 1, 3
d) 1, 2, 3

Solution: b)

Introduced in the 2016-17 kharif season, PMFBY is a central-state scheme which aims to cushion farmers against
crop loss. The central and state governments pay more than 95 per cent of the premium amount while the
farmer bears 1.5-5 per cent of the premium. As extensive usage of technology is used to settle the claims of
farmers within a stipulated time period, farmers are required to fill loss reports online which are validated by
insurance companies before the compensation amount is paid directly in their accounts.
Prior to 2020, the scheme was mandatory for farmers who availed institutional finance, but that was changed
and made voluntary for all farmers.

15) Consider the following statements.


1. Khadi and Village Industries Commission (KVIC) is an apex organisation under the Ministry of Micro,
Small, and Medium Enterprises responsible for the promotion and implementation of initiatives for the
development of Khadi and other rural industries.
2. Khadi and Village Industries Commission (KVIC) is the nodal agency for the implementation of Prime
Minister Employment Generation Program (PMEGP).
3. Scheme of Fund for Regeneration of Traditional Industries (SFURTI) aims to organize the traditional
industries and artisans into clusters to make them competitive and provide support for their long-term
sustainability.
www.insightsonindia.com 26
INSTA STATIC QUIZ
Which of the above statements is/are correct?
a) 1, 2
b) 1, 3
c) 2, 3
d) 1, 2, 3

Solution: d)

The KVIC is an apex organisation under the Ministry of Micro, Small, and Medium Enterprises responsible for
the promotion, planning, organisation, and implementation of initiatives for the development of Khadi and
other rural industries.

PMEGP is a central sector scheme administered by the Ministry of Micro, Small and Medium Enterprises
(MoMSME). Launched in 2008-09, it is a credit-linked subsidy scheme which promotes self-employment through
setting up of micro-Enterprises
Implementation:
• National Level- Khadi and Village Industries Commission (KVIC) as the nodal agency.
• State Level- State KVIC Directorates, State Khadi and Village Industries Boards (KVIBs), District Industries Centres
(DICs) and banks.

Scheme of Fund for Regeneration of Traditional Industries (SFURTI) is an initiative by Ministry of MSME to
promote Cluster development. Khadi and Village Industries Commission (KVIC) is the nodal Agency for promotion
of Cluster development for Khadi. The objective is to organize the traditional industries and artisans into clusters
to make them competitive and provide support for their long-term sustainability.

3. Geography
1) Consider the following statements:
1. Garo and Khasi hills are extensions of Purvanchals in Meghalaya forming water divide between
Brahmaputra and Barak River.
2. Rajmahal Hills are formed from rocks dating from the Jurassic Period and named after the town
of Rajmahal which lies to the east in the state of Jharkhand.
Which of the above statements is/are incorrect?
a) 1 only
b) 2 only
c) Both 1 and 2
d) Neither 1 nor 2

Solution: a)

Garo and Khasi hills are the extensions of peninsular part of the subcontinent. Along with the Karbi Anglong
plateau, the Meghalaya plateau (comprising Garo, Khasi and Jaintia hills) is separated from the Chotanagpur
plateau (part of peninsular india ) by Malda fault (in Bengal).

Rajmahal Hills are formed from rocks dating from the Jurassic Period and named after the town
of Rajmahal which lies to the east in the state of Jharkhand

2) Consider the following statements regarding Ravines in India.


1. Ravines in India are a geological feature formed millions of years ago when the peninsular plate
pressed against the Himalayas.
2. Ravines are not found in the vicinity of the Vindhyas.

www.insightsonindia.com 27
INSTA STATIC QUIZ
3. Ravines get aggravated by soil erosion.
Which of the above statements is/are correct?
a) 1, 2
b) 1, 3
c) 2, 3
d) 1, 2, 3

Solution: b)

Ravines in India are a geological feature formed millions of years ago when the peninsular plate pressed against
the Himalayas. Almost all of India’s ravines are found in the vicinity of the Vindhyas, with over 60 per cent being
in Uttar Pradesh, Madhya Pradesh, Gujarat and Rajasthan, as per the National Remote Sensing Centre,
Hyderabad.
However, ravines get aggravated by soil erosion, mostly caused by a flowing waterbody. The adjacent land gets
eroded to form narrow depressions with sharp slopes. Over time, ravines widen, rendering land uncultivable and
infertile with the erosion of the topsoil.

3) Consider the following statements:


1. The folds of the Great Himalayas are symmetrical in nature.
2. The longitudinal valley lying between lesser Himalaya and the Shiwaliks are known as Duns.
3. The part of the Himalayas lying between Sutlej and Kali rivers is known as Kumaon Himalayas.
Which of the above statements is/are correct?
a) 2, 3
b) 3 only
c) 2 only
d) 1, 2, 3

Solution: a)

The folds of the Great Himalayas are asymmetrical in nature.

The part of the Himalayas lying between Sutlej and Kali rivers is known as Kumaon Himalayas.

The longitudinal valley lying between lesser Himalaya and the Shiwaliks are known as Duns. Ex Chandigarh-Kalka
dun, Nalagarh dun, Dehra Dun, Harike dun and the Kota dun, etc. Dehra Dun is the largest of all the duns with an
approximate length of 35-45 km and a width of 22-25 km.

4) Consider the following statements about the physiography of India.


1. The core of the Great Himalayan is made up of limestone rocks.
2. The trough of the Narmada river is interposed between the Vindhyan and the Satpura ranges.
3. The Deccan Plateau receives heavy rainfall throughout the year as it falls on the windward side of
major hills of Central and Southern India.
Select the correct answer code:
www.insightsonindia.com 28
INSTA STATIC QUIZ
a) 1, 3
b) 2 only
c) 3 only
d) 1, 2, 3

Solution: b)

Statement 1: Limestones can’t bear the heavy weight of Himalayas. The core is actually made of granite rocks.
Statement 2: It is one of the rivers in India that flows in a rift valley, flowing west between the Satpura and
Vindhya ranges.
Statement 3: The Deccan Plateau is a large triangular plateau, bounded by the Vindhyas to the north and flanked
by the Eastern and Western Ghats.
This region is mostly semi-arid as it lies on the leeward side of both Ghats. Much of the Deccan is covered by
thorn scrub forest scattered with small regions of deciduous broadleaf forest. Climate in the Deccan ranges from
hot summers to mild winters.

5) Arrange the following hills of Eastern Ghats from South to North direction.
1. Nallamala hills
2. Palkonda hills
3. Javadi Hills
4. Nagari hills
Select the correct answer code:
a) 1-2-3-4
b) 2-1-3-4
c) 4-3-2-1
d) 3-4-2-1

Solution: d)

6) The ocean has several micronutrients, minerals and trace metals like cadmium or copper. Trace metals are
supplied to oceans via
1. Atmospheric deposition
2. Continental shelf interaction
3. Continental run-offs
4. Hydrothermal activities
Select the correct answer code:
a) 1, 2, 3
b) 1, 3, 4
c) 2, 3, 4
d) 1, 2, 3, 4

www.insightsonindia.com 29
INSTA STATIC QUIZ
Solution: d)

Trace metals like cadmium or copper are supplied to oceans via continental run-offs, atmospheric deposition,
hydrothermal activities and continental shelf interaction. They are essential for ocean productivity.

7) Consider the following statements regarding Brahmani river basin.


1. Brahmani is the tributary of the River Ganga.
2. Together with the river Baitarani, it forms a large delta before emptying into the Bay of Bengal.
3. It is famous for Mangrove vegetation.
Which of the above statements is/are correct?
a) 1, 2
b) 1, 3
c) 2, 3
d) 2 only

Solution: c)

Environmentalists expressed concern over the massive diversion of fresh water from the Brahmani river basin,
which could pose a grave threat to the famous mangrove vegetation in Odisha.

Together with the river Baitarani, Brahmani river forms a large delta before emptying into the Bay of Bengal.

Statement 1 is incorrect.

8) Sea level rise occurs mainly due to


1. Melting of ice sheets in Greenland and Antarctica
2. Melting of glaciers on land
3. Expansion of warm ocean waters
Select the correct answer code:
a) 1, 2
b) 2, 3
c) 1 only
d) 1, 2, 3

Solution: d)

Sea level rise occurs mainly due to the expansion of warm ocean waters, melting of glaciers on land, and the
melting of ice sheets in Greenland and Antarctica.

9) Raunthi glacier and Mrigthuni Glacier are located in


a) Uttarakhand
b) Ladakh
c) Himachal Pradesh
d) Arunachal Pradesh

Solution: a)

Mrigthuni Glacier, one of the major glaciers of Uttarakhand’s Pindar river basin originates from the 22,490-foot
Mrigthuni Peak.

10) Consider the following statements.


1. Western disturbances are periodic influxes of moisture-laden clouds from the Mediterranean that are
common during winter and cause rain in northern India.
2. The elevated temperatures and warmer waters in the Arctic Ocean reduces the intensity of Western
disturbances over North-India.
www.insightsonindia.com 30
INSTA STATIC QUIZ
Which of the above statements is/are incorrect?
a) 1 only
b) 2 only
c) Both 1 and 2
d) Neither 1 nor 2

Solution: b)

Western disturbances cause rain in northern India.


Western disturbances are periodic influxes of moisture-laden clouds from the Mediterranean that are common
during winter.

Overall elevated temperatures are also contributing to warmer waters in the Arctic Ocean and drawing colder
air from the poles with greater intensity.
This has added to the increased moisture, thereby seeding more intense western disturbance activity over north
India.

11) Consider the following statements regarding Cloudbursts.


1. Cloudbursts are very long-duration, intense rainfall events over a small area.
2. During the cloudburst, usually the relative humidity and cloud cover will be at the maximum level with
low temperature and slow winds.
Which of the above statements is/are correct?
a) 1 only
b) 2 only
c) Both 1 and 2
d) Neither 1 nor 2

Solution: b)

Cloudbursts are short-duration, intense rainfall events over a small area. According to the India Meteorological
Department (IMD), it is a weather phenomenon with unexpected precipitation exceeding 100mm/h over a
geographical region of approximately 20-30 square km.

During the cloudburst, the relative humidity and cloud cover was at the maximum level with low temperature
and slow winds.

Several studies have shown that climate change will increase the frequency and intensity of cloudbursts in many
cities across the globe.

12) Consider the following statements regarding Atlantic Meridional Overturning Circulation (AMOC).
1. The AMOC is a large system of ocean currents.
2. It distributes heat and nutrients throughout the world’s ocean basins.
3. AMOC shutdown would increase the temperature over the northern hemisphere.
Which of the above statements is/are correct?
a) 1, 2
b) 1, 3
c) 2, 3
d) 1, 2, 3

Solution: a)

Atlantic Meridional Overturning Circulation (AMOC) is a large system of ocean currents. It is the Atlantic branch
of the ocean conveyor belt or Thermohaline circulation (THC), and distributes heat and nutrients throughout the
world’s ocean basins.

www.insightsonindia.com 31
INSTA STATIC QUIZ
AMOC carries warm surface waters from the tropics towards the Northern Hemisphere, where it cools and
sinks. It then returns to the tropics and then to the South Atlantic as a bottom current. From there it is distributed
to all ocean basins via the Antarctic circumpolar current.

What happens if AMOC collapses?


Gulf Stream, a part of the AMOC, is a warm current responsible for mild climate at the Eastern coast of North
America as well as Europe. Without a proper AMOC and Gulf Stream, Europe will be very cold.
Modelling studies have shown that an AMOC shutdown would cool the northern hemisphere and decrease
rainfall over Europe. It can also have an effect on the El Nino.

13) Consider the following statements regarding Summer Solstice.


1. During summer solstice, the Earth’s axis is tilted in a way that the North Pole is tipped towards the sun
and the South Pole is away from it.
2. The amount of incoming energy the Earth received from the sun is higher at the North Pole than at the
Equator.
3. At the Arctic Circle, the sun never sets during the summer solstice.
Which of the above statements is/are correct?
a) 1, 2
b) 1, 2, 3
c) 1, 3
d) 2, 3

Solution: b)

Why do we have summer solstice?


Since Earth rotates on its axis, the Northern Hemisphere gets more direct sunlight between March and September
over the course of a day, which also means people living in the Northern Hemisphere experience summer during
this time. The rest of the year, the Southern Hemisphere gets more sunlight.

During the solstice, the Earth’s axis — around which the planet spins, completing one turn each day — is tilted
in a way that the North Pole is tipped towards the sun and the South Pole is away from it.

The solstice, as NASA puts it, is that instant in time when the North Pole points more directly toward the sun that
at any other time during the year.

The amount of light received by a specific area in the Northern Hemisphere during the summer solstice
depends on the latitudinal location of the place. The further north one moves from the equator, the more light
one receives during the summer solstice. At the Arctic Circle, the sun never sets during the solstice.

14) Consider the following statements.


1. Lightning is a very rapid and massive discharge of electricity in the atmosphere, some of which is
directed towards the Earth’s surface.
2. Lightning is generated in giant moisture-bearing clouds, whose base typically lies above 10 to 12 km
from the Earth’s surface.
Which of the above statements is/are incorrect?
a) 1 only
b) 2 only
c) Both 1 and 2
d) Neither 1 nor 2

Solution: b)

Lightning is a very rapid — and massive — discharge of electricity in the atmosphere, some of which is directed
towards the Earth’s surface. These discharges are generated in giant moisture-bearing clouds that are 10-12 km

www.insightsonindia.com 32
INSTA STATIC QUIZ
tall. The base of these clouds typically lies within 1-2 km of the Earth’s surface, while their top is 12-13 km away.
Temperatures towards the top of these clouds are in the range of minus 35 to minus 45 degrees Celsius.

15) Consider the following statements regarding Sargasso Sea.


1. The Sargasso Sea is located entirely within the Pacific Ocean.
2. It is bounded by four ocean currents forming an ocean gyre.
3. It is the only sea without a land boundary.
Which of the above statements is/are correct?
a) 2, 3
b) 3 only
c) 1, 3
d) 1, 2, 3

Solution: a)

The Sargasso Sea, located entirely within the Atlantic Ocean, is the only sea without a land boundary. It is
bounded by four ocean currents forming an ocean gyre.
It is distinguished from other parts of the Atlantic Ocean by its characteristic brown Sargassum seaweed and
often calm blue water.
The sea is bounded on the west by the Gulf Stream, on the north by the North Atlantic Current, on the east by
the Canary Current, and on the south by the North Atlantic Equatorial Current, the four together forming a
clockwise-circulating system of ocean currents termed the North Atlantic Gyre.

4. Economy
1) Phantom FDI sometimes in news is described as
a) Investments done in green field projects
b) Investments done by high net worth individuals
c) Investments that pass through empty corporate shells
d) None of the above

Solution: c)

Phantom FDI — “investments that pass through empty corporate shells” with no real business activity.

2) In an open economy without government intervention, trade deficit can be financed by


a) Foreign institutional investment
b) Monetary expansion
c) Domestic consumption
d) All of the above

Solution: a)

Total balance of payments consists of current account (includes trade, invisibles, remittances etc) as well as
capital account.
Capital inflows like FDI, FII help bridge the trade deficit and neutralize BoP.

High consumption expenditure will further inflate the import bill and cause trade deficit. And, so will monetary
expansion – pushes up demand and thus imports in the short-term aggravating the BoP.

3) A closed economy is likely to have which of the following characteristics?

www.insightsonindia.com 33
INSTA STATIC QUIZ
a) Fiscal deficit would be zero.
b) The government does not have a right to print currency.
c) The central bank does not control money supply.
d) Balance of Payments is zero.

Solution: d)

A closed economy is self-sufficient, meaning that no imports are brought in and no exports are sent out. The goal
is to provide consumers with everything that they need from within the economy's borders. A closed economy is
the opposite of an open economy, in which a country will conduct trade with outside regions.
So, if no capital or goods/services are imported, exported, the BoP will be zero.
In this case, the fiscal deficit need not be zero since a developing country may adopt expansionary fiscal policy to
tackle poverty and unemployment.

4) A Hard currency is the one which


a) is abundant in the foreign exchange market
b) is relatively stable through a short period of time with high levels of liquidity
c) does not change its value relative to movements in the international market
d) is not convertible to Special Drawing Rights (SDR)

Solution: b)

It is the international currency in which the highest faith is shown and is needed by every economy. The strongest
currency of the world is one which has a high level of liquidity, i.e. people are easily willing to sell or buy it due to
the high confidence shown in it.

5) Consider the following statements about ‘Fiat Money’.


1. It is a currency that a government has declared to be legal tender.
2. Its value increases during hyperinflation.
3. It is backed by a physical commodity.
Which of the above statements is/are incorrect?
a) 1 only
b) 2, 3
c) 1, 3
d) 1, 2, 3

Solution: b)

Fiat money is currency that a government has declared to be legal tender, but it is not backed by a
physical commodity. The value of fiat money is derived from the relationship between supply and demand rather
than the value of the material from which the money is made. Because fiat money is not linked to physical
reserves, it risks becoming worthless due to hyperinflation.

6) The Cash Reserve Ratio refers to


a) the share of Net Demand and Time Liabilities that banks have to hold as part of their cash reserves
b) the ratio of cash holding to reserves of banks
c) the share of Net Demand and Time Liabilities that banks have to hold as cash deposit with the RBI
d) the share of Net Demand and Time Liabilities that banks have to hold as liquid assets

Solution: c)

Cash Reserve Ratio refers to the fraction of the total Net Demand and Time Liabilities (NDTL) of a Scheduled
Commercial Bank held in India, that it has to maintain as cash deposit with the Reserve Bank of India (RBI).

www.insightsonindia.com 34
INSTA STATIC QUIZ
It shall apply to all Scheduled Commercial Banks (SCBs) (including Regional Rural Banks), Small Finance Banks
(SFBs), Payments Banks, Local Area Banks (LABs), Primary (Urban) Co-operative Banks (UCBs), State Co-operative
Banks (StCBs) and District Central Co-operative Banks (DCCBs) unless stated to the contrary.

7) Consider the following statements.


1. Nominal GDP is calculated in a way such that the goods and services are evaluated at some constant
set of prices.
2. If the Real GDP changes, it implies that the volume of production is undergoing changes.
3. The ratio of nominal GDP to real GDP gives us an idea of how the prices have moved from the base year
to the current year.
Which of the above statements is/are correct?
a) 1, 2
b) 1, 3
c) 2, 3
d) 1, 2, 3

Solution: c)

In order to compare the GDP figures (and other macroeconomic variables) of different countries or to compare
the GDP figures of the same country at different points of time, we cannot rely on GDPs evaluated at current
market prices. For comparison we take the help of real GDP. Real GDP is calculated in a way such that the goods
and services are evaluated at some constant set of prices (or constant prices).
Since these prices remain fixed, if the Real GDP changes, we can be sure that it is the volume of production
which is undergoing changes. Nominal GDP, on the other hand, is simply the value of GDP at the current
prevailing prices.

Notice that the ratio of nominal GDP to real GDP gives us an idea of how the prices have moved from the base
year (the year whose prices are being used to calculate the real GDP) to the current year.
The ratio of nominal to real GDP is a well-known index of prices. This is called GDP Deflator.

8) Consider the following statements regarding Open Market Operations.


1. Open Market Operations refers to buying and selling of bonds issued by the Government in the open
market.
2. Selling of a bond by RBI leads to increases in the total amount of reserves in the economy and thus
increases the money supply.
Which of the above statements is/are incorrect?
a) 1 only
b) 2 only
c) Both 1 and 2
d) Neither 1 nor 2

Solution: b)

Open Market Operations refers to buying and selling of bonds issued by the Government in the open
market. This purchase and sale is entrusted to the Central bank on behalf of the Government. When RBI buys a
Government bond in the open market, it pays for it by giving a cheque. This cheque increases the total amount of
reserves in the economy and thus increases the money supply. Selling of a bond by RBI (to private individuals or
institutions) leads to reduction in quantity of reserves and hence the money supply.

9) Which of the following is not a consequence of rupee depreciation?


a) Exports will become competitive
b) Increase in the cost of imports
c) Reduction in remittances from abroad
d) Foreign trips will become costlier

www.insightsonindia.com 35
INSTA STATIC QUIZ
Solution: c)

Rupee deprecation badly affects importers or those who wish to visit foreign countries for holidays as they need
more local currency to get the same service or product.
When rupee depreciates exporters from India are benefited. (Eg: Software companies, seafood exporters etc.)
Depreciation of rupee benefit the overseas Indians as those who are working abroad will gain more on
remitting money to their homeland.

10) Consider the following statements


1. Bonds are papers bearing the promise of a future stream of monetary returns over a certain period of
time.
2. The speculative demand for money is directly related to the rate of interest.
Which of the above statements is/are correct?
a) 1 only
b) 2 only
c) Both 1 and 2
d) Neither 1 nor 2

Solution: a)

An individual may hold her wealth in the form of landed property, bullion, bonds, money etc. For simplicity, let us
club all forms of assets other than money together into a single category called bonds. Typically, bonds are
papers bearing the promise of a future stream of monetary returns over a certain period of time. These papers
are issued by governments or firms for borrowing money from the public and they are tradable in the market.
When the interest rate is very high everyone expects it to fall in future and hence anticipates capital gains from
bond-holding. Hence people convert their money into bonds. Thus, speculative demand for money is low. When
interest rate comes down, more and more people expect it to rise in the future and anticipate capital loss. Thus,
they convert their bonds into money giving rise to a high speculative demand for money. Hence speculative
demand for money is inversely related to the rate of interest.

11) Consider the following statements regarding Commercial Paper (CP).


1. Commercial Paper (CP) is a secured money market instrument issued in the form of a promissory note.
2. CP will be issued at a discount to face value as may be determined by the issuer.
3. All the corporates are automatically eligible to issue CP.
Which of the above statements is/are correct?
a) 1, 2
b) 2 only
c) 2, 3
d) 1, 3

Solution: b)

Commercial Paper (CP) is an unsecured money market instrument issued in the form of a promissory note.
CP will be issued at a discount to face value as may be determined by the issuer.

The following conditions have to be fulfilled by corporates to receive privileges for issuing commercial paper:
• The tangible net worth of the company should not be less than 4 Crores, as per the latest audited Balance-
Sheet.
• The companies should have the ‘sanctioned working capital limit’ by the banks or any Financial Institutions
(FIs).
• The Financial Institutions or Banks should classify the ‘Borrowal Account’ as a Standard asset.

12) Consider the following statements regarding Rahn Curve.


a) It describes how household expenditure on a particular good or service varies with household income.
b) It is a graphical representation of the relationship between unemployment and the job vacancy rate.
www.insightsonindia.com 36
INSTA STATIC QUIZ
c) It is a graph which indicates that there is a level of government spending that maximizes economic
growth.
d) It is a method for predicting the likely production rate of any finite resource over time.

Solution: c)

The Rahn curve is a graph used to illustrate an economic theory, proposed in 1996 by American economist
Richard W. Rahn, which indicates that there is a level of government spending that maximizes economic growth.
The theory is used by classical liberals to argue for a decrease in overall government spending and taxation. The
inverted-U-shaped curve suggests that the optimal level of government spending is 15–25% of GDP.

13) The term Stagflation is often seen in news. The conditions associated with Stagflation are
1. Inflation
2. Low economic growth
3. High employment
Select the correct answer code:
a) 2, 3
b) 1, 2
c) 1, 3
d) 1, 2, 3

Solution: b)

Stagflation is a condition of slow economic growth and relatively high unemployment, or economic stagnation,
accompanied by rising prices, or inflation. It can also be defined as inflation and a decline in gross domestic
product (GDP).

14) Which of the following statements best describes the term Mezzanine financing?
a) It is a financial instrument used by companies for short term loans.
b) It is a financing mechanism used by venture capitals to invest in high growth start-ups.
c) It is a financial instrument which is a hybrid of debt and equity financing
d) None of the above

Solution: c)

Mezzanine financing is a hybrid of debt and equity financing that gives the lender the right to convert the debt
to an equity interest in the company in case of default, generally, after venture capital companies and other
senior lenders are paid. In terms of risk, it exists between senior debt and equity.

15) Consider the following statements regarding Factor Income.


1. Factor income is income received from the factors of production.
2. The factor income of all normal residents of a country is referred to as the national income.
3. It can be used to expose disparities in income distribution.
www.insightsonindia.com 37
INSTA STATIC QUIZ
Which of the above statements is/are correct?
a) 1, 2
b) 1, 3
c) 2, 3
d) 1, 2, 3

Solution: d)

Factor income is the flow of income that is derived from the factors of production—the general inputs required
to produce goods and services.
Factor income on the use of land is called rent, income generated from labor is called wages, and income
generated from capital is called profit.
The factor income of all normal residents of a country is referred to as the national income, while factor income
and current transfers together are referred to as private income.
Factor income is most commonly used in macroeconomic analysis, helping governments to determine the
difference between gross domestic product (GDP) and gross national product (GNP).
It can also be used to expose disparities in income distribution.

5. Art and Culture


1) Consider the following statements:
1. They developed Vesara style of architecture.
2. Their structural temples exist at Aihole, Badami and Pattadakal.
3. Their administration was highly centralized.
The above statements most accurately refer to?
a) Cholas
b) Chalukyas
c) Cheras
d) Pandyas

Solution: b)

Administration: The Chalukya administration was highly centralized unlike that of the Pallavas and the Cholas.
Village autonomy was absent under the Chalukyas.

Architecture: The Chalukyas were great patrons of art. They developed the vesara style in the building of
structural temples. However, the vesara style reached its culmination only under the Rashtrakutas and the
Hoysalas.
Cave temple architecture was also famous under the Chalukyas. Their cave temples are found in Ajanta, Ellora
and Nasik.
The best specimens of Chalukya paintings can be seen in the Badami.

2) Consider the following statements regarding Kalbelia.


1. It is a folk dance performed by the women of the Kalbelia community of Gujarat.
2. ‘Been’ is the popular musical instrument of this dance form.
3. The UNESCO has inscribed Kalbelia folk songs and dances in the Representative List of the Intangible
Cultural Heritage of Humanity.
Which of the above statements is/are correct?
a) 1, 2
b) 2, 3
c) 1, 3

www.insightsonindia.com 38
INSTA STATIC QUIZ
d) 1, 2, 3

Solution: b)

Kalbelia is a sensuous folk dance performed by the women of the Kalbelia community of Rajasthan. The
costumes and dance movement are similar to that of the serpents. ‘Been’ (wind instrument played by snake
charmers) is the popular musical instrument of this dance form. The UNESCO has inscribed Kalbelia folk songs
and dances in the Representative List of the Intangible Cultural Heritage of Humanity in 2010.

3) Which of the following are major features of Chola age paintings?


1. They were often done on temple panels.
2. They show narrations and aspects related to Lord Shiva.
3. They avoid depictions of humans and plants.
Select the correct answer code:
a) 1, 2
b) 2, 3
c) 1, 3
d) 1, 2, 3

Solution: a)

Chola Paintings included humans and plants also.

4) Consider the following statements regarding the temple architecture of Kashmir.


1. Kashmir’s proximity to prominent Gandhara sites lent the region a strong Gandhara influence.
2. The Karkota period of Kashmir is most significant in terms of architecture.
3. There was a tradition of a water tank attached to the shrine.
Which of the above statements is/are correct?
a) 1, 2
b) 2, 3
c) 1, 3
d) 1, 2, 3

Solution: d)

Kashmir’s proximity to prominent Gandhara sites (such as Taxila, Peshawar and the northwest frontier) lent the
region a strong Gandhara influence by the fifth century CE.

The Karkota period of Kashmir is the most significant in terms of architecture. One of the most important
temples is Pandrethan, built during the eighth and ninth centuries. In keeping with the tradition of a water tank
attached to the shrine, this temple is built on a plinth built in the middle of a tank. Although there are evidences
of both Hindu and Buddhist followings in Kashmir, this temple is a Hindu one, possibly dedicated to Shiva. The
architecture of this temple is in keeping with the age-old Kashmiri tradition of wooden buildings.

5) Consider the following statements.


1. This Veda is also known as Brahma Veda
2. It is mostly concerned with peace and prosperity of the human society
3. It focuses on treatment of several ailments
The above statements refer to
a) Rig Veda
b) Sama Veda
c) Atharva Veda
d) Yajur Veda

www.insightsonindia.com 39
INSTA STATIC QUIZ
Solution: c)

Atharva Veda:
This Veda is also known as Brahma Veda and has been attributed to two rishis called Atharvah and Angira,
respectively. While it is mostly concerned with peace and prosperity of the human society and covers all aspects
of a man’s daily life, it specifically focuses on treatment of several ailments. The book is known to prescribe
treatment for almost 99 diseases.

6) Consider the following statements regarding Vedanta School of Thought.


1. This school upholds the philosophies of life as elaborated in the Upanishads.
2. The Vedanta theory denies the theory of rebirth.
Which of the above statements is/are correct?
a) 1 only
b) 2 only
c) Both 1 and 2
d) Neither 1 nor 2

Solution: a)

Vedanta is made of two words- ‘Veda’ and ‘ant’, i.e. the end of the Vedas. This school upholds the philosophies
of life as elaborated in the Upanishads.
This argument considers atma and brahma to be the same and if a person attains the knowledge of the self, he
would automatically understand brahma and would achieve salvation. This argument would make brahma and
atma indestructible and eternal.
The Vedanta theory also gives credence to the Theory of Karma. The theory believes in Punarjanama or rebirth.
They also argued that a person would have to bear the brunt of their actions from the previous birth in the next
one.

7) Consider the following statements regarding Mahajanapadas.


1. Early Buddhist and Jaina texts mention sixteen states known as mahajanapadas.
2. All the mahajanapadas were ruled by kings.
3. Each mahajanapada had a capital city, which was often fortified.
Which of the above statements is/are correct?
a) 1, 2
b) 1, 3
c) 2, 3
d) 1, 2, 3

Solution: b)

Early Buddhist and Jaina texts mention, amongst other things, sixteen states known
as mahajanapadas. Although the lists vary, some names such as Vajji, Magadha, Koshala, Kuru, Panchala,
Gandhara and Avanti occur frequently. Clearly, these were amongst the most important mahajanapadas.

While most mahajanapadas were ruled by kings, some, known as ganas or sanghas, were oligarchies, where
power was shared by a number of men, often collectively called rajas. Both Mahavira and the Buddha belonged
to such ganas. In some instances, as in the case of the Vajji sangha, the rajas probably controlled resources such
as land collectively. Although their histories are often difficult to reconstruct due to the lack of sources, some of
these states lasted for nearly a thousand years.

Each mahajanapada had a capital city, which was often fortified. Maintaining these fortified cities as well as
providing for incipient armies and bureaucracies required resources.

8) Which of the following are considered as Triratnas under Jainism?


1. Right livelihood
www.insightsonindia.com 40
INSTA STATIC QUIZ
2. Right knowledge
3. Right conduct
4. Right speech
5. Right faith
Select the correct answer code:
a) 1, 2, 4
b) 1, 3, 4
c) 2, 3, 5
d) 1, 3, 5

Solution: c)

The three principles of Jainism, also known as Triratnas (three gems), are:
• right faith
• right knowledge
• right conduct.
Right faith is the belief in the teachings and wisdom of Mahavira. Right Knowledge is the acceptance of the theory
that there is no God and that the world has been existing without a creator and that all objects possess a soul.
Right conduct refers to the observance of the five great vows:
• not to injure life
• not to lie
• not to steal
• not to acquire property
• not to lead an immoral life.

9) Consider the following statements regarding the teachings of Mahavira.


1. Mahavira regarded all objects, both animate and inanimate, have souls and various degrees of
consciousness.
2. Mahavira rejected the authority of the Vedas and objected to the Vedic rituals.
3. He considered the practice of agriculture as the purest and credible occupation.
Which of the above statements is/are correct?
a) 1 only
b) 1, 2
c) 1, 3
d) 2, 3

Solution: b)

Mahavira regarded all objects, both animate and inanimate, have souls and various degrees of consciousness.
They possess life and feel pain when they are injured.
Mahavira rejected the authority of the Vedas and objected to the Vedic rituals. He advocated a very holy and
ethical code of life. Even the practice of agriculture was considered sinful as it causes injury to the earth, worms
and animals.

10) Consider the following statements regarding Vesara School of architecture.


1. In the Vesara School of architecture, the pillars, doorways and the ceilings were decorated with
intricate carvings.
2. The Influence of Nagara style is in intricate carvings and sculptures, design of Vimana and Step or
terraced Shikara of Vesara temples.
3. The Influence of Dravida style is seen in Curvilinear Shikhara and square base of Vesara temples.
Which of the above statements is/are correct?
a) 1, 2
b) 1 only
c) 1, 3

www.insightsonindia.com 41
INSTA STATIC QUIZ
d) 1, 2, 3

Solution: b)

Also known as the Karnataka school of architecture, it was conceptualized under the later Chalukya rulers in the
mid-seventh century A.D. It combined features of both Nagara school and Dravidian school and resulted in
a hybridised style. Some of its features are:
• Emphasis on vimana and mandapa
• Open ambulatory passageway
• The pillars, doorways and the ceilings were decorated with intricate carvings.

Influence of Nagara style is in Curvilinear Shikhara and square base of Vesara temples.

Influence of Dravida style is seen in intricate carvings and sculptures, design of Vimana and Step or terraced
Shikara of Vesara temples.

Example: Doddabasappa temple at Dambal, Ladkhan temple at Aihole, temples at Badami etc.

6. History
1) The government of India had to make huge payments to people in England, which were called ‘Home
Charges’. They included:
1. Interest on public debt raised in England
2. Annuities on account of railway and irrigation works
3. Payments in connection with civil departments where Englishmen were employed.
4. India Office expenses including pensions to retired officials who had worked for India in England.
Which of the above statements is/are correct?
a) 1, 2, 3
c) 1, 2, 4
c) 2, 3, 4
d) 1, 2, 3, 4

Solution: d)

The government of India had to make huge payments to people in England on account of political, administrative
and commercial connections established between India and England. These commitments were called ‘Home
Charges’. They included:
• Interest on public debt raised in England at comparatively higher rates;
• Annuities on account of railway and irrigation works;
• Payments in connection with civil departments where Englishmen were employed;
• India Office expenses including pensions to retired officials who had worked in India or who had
worked for India in England and retired there, pensions to army and naval personnel, and their furlough
allowances.

2) Match the following organization with their founder.


1. Social Service League: Gopalakrishna Gokhale
2. Servants of India Society: N M Joshi
3. Indian National Social Conference: M G Ranade
Select the correct answer code:
a) 1, 3
b) 2, 3

www.insightsonindia.com 42
INSTA STATIC QUIZ
c) 3 only
d) 1, 2

Solution: c)

Social Service League: 1911 Bombay: N M Joshi: Its aim was to secure for the masses better and reasonable
conditions of life and work.

Servants of India Society: 1905 Bombay: Gopalakrishna Gokhale: Its aim was to train Indians in different fields
for the service of their motherland.

Indian National Social Conference: 1887 Bombay: M G Ranade: Its aim was to remove the social evils prevalent in
the Indian society and to promote the welfare of women.

3) Consider the following statements regarding Desai-Liaqat pact.


1. It recommended for an equal number of persons nominated by the Congress and the Muslim League in
the central legislature.
2. It recommended 50% reserved seats for minorities.
Which of the above statements is/are correct?
a) 1 only
b) 2 only
c) Both 1 and 2
d) Neither 1 nor 2

Solution: a)

Bhulabhai Desai, leader of the Congress Party in the Central Legislative Assembly, met Liaqat Ali Khan, deputy
leader of the Muslim League in that Assembly, and both of them came up with the draft proposal for the
formation of an interim government at the centre, consisting of:
• An equal number of persons nominated by the Congress and the League in the central legislature.
• 20% reserved seats for minorities.
No settlement could be reached between the Congress and the League on these lines.

4) The Zimmerman Plan is related to


a) design the future constitution of independent India.
b) Improve the relationship between British government and princely states
c) Indo-German plan to bring about an all-India insurrection against British government.
d) Plan of British government to provide Fundamental Rights to Indian citizens.

Solution: c)

The Zimmerman Plan is an Indo-German plan which aimed to bring about an all-India insurrection against
British government during the time of World war I.

5) Clement Attlee’s statement got lot of importance in India’s Freedom Struggle. What is the significance of it?
a) Release of political prisoners of Freedom struggle
b) It declared the British intention of leaving the Indian subcontinent.
c) It transferred the power completely to the princely states.
d) None of the above

Solution: b)

Clement Attlee, the British prime minister, sensing the trouble all around, made an announcement on February
20, 1947. The British House of Commons declared the British intention of leaving the Indian subcontinent.

www.insightsonindia.com 43
INSTA STATIC QUIZ
A deadline of June 30, 1948 was fixed for transfer of power even if the Indian politicians had not agreed by that
time on the constitution.

6) Which of the following food grains were found extensively in Harappan civilization?
1. Wheat
2. Rice
3. Barley
4. Chickpea
Select the correct answer code:
a) 1, 3, 4
b) 1, 2
c) 2, 3, 4
d) 1, 2, 3, 4

Solution: a)

The Harappans ate a wide range of plant and animal products, including fish. Archaeologists have been able to
reconstruct dietary practices from finds of charred grains and seeds. Grains found at Harappan sites include
wheat, barley, lentil, chickpea and sesame. Millets are found from sites in Gujarat. Finds of rice are relatively
rare.

7) Which of the following statements regarding the medieval Indian rulers is correct?
a) Muhammad Bin Tughlaq was succeeded by his uncle to the military.
b) Balban introduced the branding of horses in his military.
c) Firoz Shah Tughlaq set up a separate department of slaves.
d) Alauddin Khalji first set up a separate ariz's department.

Solution: c)

Firoz Shah Tughlaq established a new department called Diwan-i- Khairat (Department of Charity) to take care of
orphans and widows. Free hospitals like Dar-ul-Shafa and marriage bureaus for poor Muslims were also
established. He has also created the department of slaves or Diwan-i-Bandagan.

Muhammad bin Tughlaq was succeeded by his cousin (not uncle) Firoz Shah Tughlaq.

Ala-ud-din Khalji introduced a system of chehra, an identity card system for every soldier, Dagh to brand horses
to be used specifically for wars. Dagh and chehra were the 2 reforms in the army.

Diwan-i-Arz was established by Balban. Diwan-i-Arz was essentially the Department of Military managed by Ariz-
i-Mamalik. He was accountable for the regulation and preservation of the royal army.

8) The Chauth and Sardeshmukhi were taxes conceived during the times of
a) Madhavrao
b) Shivaji
c) Balaji Bajirao
d) Jahangir

Solution: b)

Chauth and Sardeshmukhi were taxes conceived during the times of the Shivaji Maharaj.
‘Chauth’ means basically 1/4th i.e 25% of gross revenue or produce to be paid to jagirdars of Maratha Empire
from hostile or alien state. In return the state paying the tax would get an assurance of non-aggression from the
Maratha army against the enemy state.
‘Sardeshmukhi’ is an additional 10% tax levied upon the collected ‘Chauth’. The reasons for the additional tax was
due to the King claiming hereditary rights upon the tax collection.
www.insightsonindia.com 44
INSTA STATIC QUIZ

9) Which of the following were the achievements of Swarajist Activity in Councils?


1. They exposed the hollowness of the Montford scheme.
2. Vithalbhai Patel was elected speaker of Central Legislative Assembly.
3. Defeat of the Public Safety Bill.
Select the correct answer code:
a) 1, 2
b) 1, 3
c) 2, 3
d) 1, 2, 3

Solution: d)

Swarajist Activity in Councils


Achievements
(i) With coalition partners, they out-voted the government several times, even on matters relating to budgetary
grants, and passed adjournment motions.
(ii) They agitated through powerful speeches on self-government, civil liberties and industrialisation.
(iii) Vithalbhai Patel was elected speaker of Central Legislative Assembly in 1925.
(iv) A noteworthy achievement was the defeat of the Public Safety Bill in 1928 which was aimed at empowering
the Government to deport undesirable and subversive foreigners (because the Government was alarmed by the
spread of socialist and communist ideas and believed that a crucial role was being played by the British and other
foreign activists).
(v) By their activities, they filled the political vacuum at a time when the national movement was recouping its
strength.
(vi) They exposed the hollowness of the Montford scheme.
(vii) They demonstrated that the councils could be used creatively.

10) Which of the following were the features of Government of India Act, 1919?
1. Women were given the right to vote.
2. A bicameral arrangement was introduced.
3. The governor could veto bills and issue ordinances.
4. The governor-general retained full control over the “reserved” subjects in the provinces.
Select the correct answer code:
a) 1, 2, 3
b) 1, 3, 4
c) 2, 3, 4
d) 1, 2, 3, 4

Solution: d)

The main features of the Montford Reforms


(1) Provincial Government—Introduction of Dyarchy:
(a) Executive:
• Dyarchy, i.e., rule of two—executive councillors and popular ministers—was introduced. The
governor was to be the executive head in the province.
• Subjects were divided into two lists: “reserved” which included subjects such as law and order,
finance, land revenue, irrigation, etc., and “transferred” subjects such as education, health, local
government, industry, agriculture, excise, etc.
• The “reserved” subjects were to be administered by the governor through his executive council of
bureaucrats, and the “transferred” subjects were to be administered by ministers nominated
from among the elected members of the legislative council.
• The ministers were to be responsible to the legislature and had to resign if a no-confidence
motion was passed against them by the legislature, while the executive councillors were not to be
responsible to the legislature.
www.insightsonindia.com 45
INSTA STATIC QUIZ
• In case of failure of constitutional machinery in the province the governor could take over the
administration of “transferred” subjects also.
• The secretary of state and the governor-general could interfere in respect of “reserved” subjects
while in respect of the “transferred” subjects; the scope for their interference was restricted.

(b) Legislature:
• Provincial Legislative Councils were further expanded—70% of the members were to be elected.
• The system of communal and class electorates was further consolidated.
• Women were also given the right to vote.
• The Legislative Councils could initiate legislation but the governor’s assent was required. The
governor could veto bills and issue ordinances.
• The Legislative Councils could reject the budget but the governor could restore it, if necessary.
• The legislators enjoyed freedom of speech.

(2) Central Government—Still Without Responsible Government:


(a) Executive:
• The governor-general was to be the chief executive authority.
• There were to be two lists for administration– central and provincial.
• In the viceroy’s executive council of 8, three were to be Indians.
• The governor-general retained full control over the “reserved” subjects in the provinces.
• The governor-general could restore cuts in grants, certify bills rejected by the Central Legislature,
summon, prorogue, dissolve the Chambers, and issue ordinances.

(b) Legislature:
• A bicameral arrangement was introduced.
• The Council of State had tenure of 5 years and had only male members, while the Central
Legislative Assembly had tenure of 3 years.
• The legislators could ask questions and supplementaries pass adjournment motions and vote a
part of the budget, but 75% of the budget was still not votable.
• Some Indians found their way into important committees including finance.

11) Consider the following statements regarding Ryotwari System.


1. It was introduced by Thomas Munro.
2. The ownership rights were handed over to the peasants.
3. Major areas of introduction include Madras, Bombay, parts of Assam and Coorg provinces of British
India.
Which of the above statements is/are correct?
a) 1, 2
b) 1, 3
c) 2, 3
d) 1, 2, 3

Solution: d)

The Ryotwari system was a land revenue system in British India, introduced by Sir Thomas Munro in 1820 based
on system administered by Captain Alexander Read. This system was exactly opposite to the Zamindari system.
In this system, peasants were given the ownership. It was first introduced in Madras presidency. It was later
extended to Bombay, Parts of Bengal, Assam, Coorg etc.

12) The Ilbert Bill Controversy is said to be a high watermark in the history of Indian National Movement. This is
because it invoked issues of
1. Racial discrimination between Indian and Europeans
2. Suppression of Indian media houses and their nationalization by the Government
www.insightsonindia.com 46
INSTA STATIC QUIZ
3. Security lapse on the Indian borders, especially the North-Western frontier
Select the correct answer code:
a) 1 only
b) 1, 2
c) 2, 3
d) 1, 3

Solution: a)

Lord Ripon wanted to remove two kinds of law that had been prevalent in India. According to the system of law, a
European could be tried only by a European Judge or a European Magistrate.
The disqualification was unjust and it was sought to cast a needless discredit and dishonour upon the Indian-born
members of the judiciary.
C.P. Ilbert, Law Member, introduced a bill in 1883 to abolish this discrimination in judiciary. But Europeans
opposed this Bill strongly. They even raised a fund of one lakh fifty thousand rupees and established an
organisation called the Defence Association.
They also suggested that it was better to end the English rule in India than to allow the English to be subjected to
the Indian Judges and Magistrates. The press in England joined the issue. Hence, Ripon amended the bill to satisfy
the English in India and England.

13) Partition of Bengal was revoked in 1911 by


a) Lord Minto
b) Lord Hardinge
c) Lord Chelmsford
d) Lord Curzon

Solution: b)

In July 1905, Curzon announced the partition of the undivided Bengal Presidency. Curzon left for Britain in 1905,
but the agitation continued for many years. Partition was finally reversed in 1911 by Lord Hardinge in the face of
unrelenting opposition.

14) In 1720, the British government enacted the Calico Act. What is it related to?
a) Disallowing Indians from entering Civil Services
b) Barring of company officials from private trade
c) Banning the use of printed cotton textile imported from India.
d) Compulsory use of English language in Company affairs

Solution: c)

The Calico Act banned the import of most cotton textiles into England, followed by the restriction of sale of
most cotton textiles. It was a form of economic protectionism, largely in response to India (particularly Bengal),
which dominated world cotton textile markets at the time. The Act was a precursor to the Industrial Revolution,
when Britain eventually surpassed India as the world's leading textile manufacturer in the 19th century.

15) The Wavell Plan, arrived at the Simla Conference 1945 provided for which of the following?
1. Partition of India
2. Removing any caste and religion-based quota in the Executive Council
3. Indianization of the Viceroy’s Executive Council
Select the correct answer code:
a) 1, 2
b) 2, 3
c) 3 only
d) 1, 2, 3

www.insightsonindia.com 47
INSTA STATIC QUIZ
Solution: c)

As per the Plan, all the members of the Council, except the Viceroy and the Commander-in-Chief, would be
Indians. It said, in the Council there would be equal representation of caste Hindus and Muslims. It proposed for
a future constitution of India, not its partition.

7. Environment
1) Which of the following best defines the term ‘Biosphere’?
a) It is the abiotic component of all forms on earth.
b) It is the narrow zone where land, water and air come together to contain life forms.
c) It is the upper region of lithosphere.
d) It is the lower atmospheric region near the hydrosphere.

Solution: b)

The solid portion of the earth on which we live is called the Lithosphere.

The gaseous layers that surround the earth, is the Atmosphere, where oxygen, nitrogen, carbon dioxide and other
gases are found.

Water covers a very big area of the earth’s surface and this area is called the Hydrosphere. The Hydrosphere
comprises water in all its forms, that is, ice, water and water vapour.

The Biosphere is the narrow zone where we find land, water and air together, which contains all forms of life.

2) Which of the following are the Common characteristics of Invasive alien species.
1. High dispersal ability
2. Phenotypic plasticity
3. Rapid reproduction and growth.
4. Ability to survive in a wide range of environmental conditions.
Which of the above statements is/are correct?
a) 1, 2, 3
b) 1, 3, 4
c) 2, 3, 4
d) 1, 2, 3, 4

Solution: d)

Common characteristics of Invasive alien species include rapid reproduction and growth, high dispersal ability,
phenotypic plasticity (ability to adapt physiologically to new conditions), and ability to survive on various food
types and in a wide range of environmental conditions. A good predictor of invasiveness is whether a species has
successfully or unsuccessfully invaded elsewhere.

3) Consider the following statements regarding Biotope.


1. It is an ecological area that is usually larger than an ecosystem.
2. It supports a particular range of biological communities.
3. It is a common practice to isolate biotopes from each other for niche biodiversity propagation.
Which of the above statements is/are correct?
a) 1, 2
b) 2 only
c) 2, 3

www.insightsonindia.com 48
INSTA STATIC QUIZ
d) 1, 3

Solution: b)

Biotope is an ecological area that supports a particular range of biological communities.


Biotope is almost synonymous with the term habitat.
A biotope is generally not considered to be a large-scale phenomenon. For example, a biotope might be a
neighbouring park, a back garden, even potted plants or a fish tank.
In other words, the biotope is not a macroscopic but a microscopic approach to preserving the ecosystem and
biological diversity.

It is commonly emphasised that biotopes should not be isolated. Instead biotopes need to be connected to each
other and other surrounding life for without these connections to life-forms such as animals and plants, biotopes
would not effectively work as a place in which diverse organisms live.
So one of the most effective strategies for regenerating biotopes is to plan a stretch of biotopes, not just a point
where animals and plants come and go.

4) “A large tree shades a small plant, retarding the growth of the small plant. The
small plant has no effect on the large tree” this example is related to which type of biotic interaction?
a) Commensalism
b) Amensalism
c) Competition
d) Neutralism

Solution: b)

Types of biotic interaction:


Mutualism: both species benefit, Example: in pollination mutualisms, the pollinator gets food (pollen, nectar), and
the plant has its pollen transferred to other flowers for cross-fertilization (reproduction).

Commensalism: one species benefits, the other is unaffected. Example: cow dung provides food and shelter to
dung beetles. The beetles have no effect on the cows.

Competition: both species are harmed by the interaction. Example: if two species eat the same food, and there
isn't enough for both, both may have access to less food than they would if alone. They both suffer a shortage of
food

Predation and parasitism: one species benefits, the other are harmed. Example: predation-one fish kills and eats
parasitism: tick gains benefit by sucking blood; host is harmed by losing blood.

Amensalism: One species is harmed, the other is unaffected. Example: A large tree shades a small plant,
retarding the growth of the small plant. The small plant has no effect on the large tree.

5) Consider the following


1. Soil
2. Non- Green plants
3. Water
4. Decomposers
Which of the above components is/are Biotic components of Environment?
a) 1, 2
b) 2, 3
c) 2, 4
d) 3, 4

www.insightsonindia.com 49
INSTA STATIC QUIZ
Solution: c)

Everything that surrounds or affects an organism during its life time is collectively known as its environment
which comprises both living (biotic) and non-living (abiotic) components. The environment is not static. Both
biotic and abiotic factors are in a flux and keep changing continuously.

6) Consider the following statements regarding Bacteriophages.


1. A Bacteriophage is a type of bacteria that can kill viruses.
2. Bacteriophages are among the most common and diverse entities in the biosphere.
3. They are seen as a possible therapy against multi-drug-resistant strains of many bacteria.
Which of the above statements is/are correct?
a) 1, 2
b) 2, 3
c) 1, 3
d) 1, 2, 3

Solution: b)

• Bacteriophages are the viruses that can kill the bacteria.


• They are comprised of a protein capsule around an RNA or DNA genome.
• They are ubiquitous viruses, found wherever bacteria exist.
• They are seen as a possible therapy against multi-drug-resistant strains of many bacteria.

7) Which of the following regions are considered as tiger landscapes in India?


1. Sundarbans
2. Shivalik Hills and Gangetic Plains
3. North-East Hills and Brahmaputra Plains
4. Central Indian Landscape and Eastern Ghats
Select the correct answer code:
a) 1, 2, 3
b) 1, 3, 4
c) 1, 2, 3, 4
d) 2, 3, 4

Solution: c)

India’s five tiger landscapes are: Shivalik Hills and Gangetic Plains, Central Indian Landscape and Eastern Ghats,
Western Ghats, North-East Hills and Brahmaputra Plains, and the Sundarbans.

8) Which of the following methods can help soil store more carbon?
1. Use of organic fertilizers and compost

www.insightsonindia.com 50
INSTA STATIC QUIZ
2. Intercropping and growing more grass strips
3. Extensive grazing
4. Never leaving soil bare
Select the correct answer code:
a) 1, 2, 3
b) 1, 3, 4
c) 1, 2, 3, 4
d) 1, 2, 4

Solution: d)

9) Consider the following statements regarding Urban heat island (UHI).


1. An urban heat island (UHI) is an urban area that is significantly warmer than its surrounding rural areas
due to human activities.
2. The UHI decreases air quality by increasing the production of pollutants such as ozone, and decreases
water quality.
3. The effect of Urban heat island is more severe during daytime than in night time.
Which of the above statements is/are correct?
a) 1, 2
b) 1, 3
c) 2, 3
d) 1, 2, 3

Solution: a)

An urban heat island (UHI) is an urban area or metropolitan area that is significantly warmer than its
surrounding rural areas due to human activities. The temperature difference is usually larger at night than
during the day, and is most apparent when winds are weak. UHI is most noticeable during the summer and
winter. The main cause of the urban heat island effect is from the modification of land surfaces.

The UHI decreases air quality by increasing the production of pollutants such as ozone, and decreases water
quality as warmer waters flow into area streams and put stress on their ecosystems.

www.insightsonindia.com 51
INSTA STATIC QUIZ

Mitigation of the urban heat island effect can be accomplished through the use of green roofs and the use of
lighter-colored surfaces in urban areas, which reflect more sunlight and absorb less heat.
Concerns have been raised about possible contribution from urban heat islands to global warming.

10) Which of the following are the Tiger Reserves located in Arunachal Pradesh?
1. Kamlang Tiger Reserve
2. Nameri Tiger Reserve
3. Namdhapa Tiger Reserve
4. Pakke Tiger Reserve
Select the correct answer code:
a) 1, 2, 3
b) 1, 3, 4
c) 1, 2, 4
d) 1, 2, 3, 4

Solution: b)

Nameri Tiger Reserve is in Assam.

11) Consider the following statements.


1. Decomposition is the process where decomposers break down complex organic matter into inorganic
substances like carbon dioxide, water and nutrients.
2. Catabolism is the process where the humus is further degraded by some microbes and release of
inorganic nutrients occur.
3. Mineralization is process where bacterial and fungal enzymes degrade detritus into simpler inorganic
substances.
Which of the above statements is/are correct?
a) 1 only
b) 1, 3
c) 2, 3
d) 1, 2, 3

Solution: a)

• You may have heard of the earthworm being referred to as the farmer’s ‘friend’. This is so because they help
in the breakdown of complex organic matter as well as in loosening of the soil. Similarly, decomposers break
down complex organic matter into inorganic substances like carbon dioxide, water and nutrients and the
process is called decomposition. Dead plant remains such as leaves, bark, flowers and dead remains of
animals, including fecal matter, constitute detritus, which is the raw material for decomposition. The
important steps in the process of decomposition are fragmentation, leaching, catabolism, humification and
mineralisation.
• Detritivores (e.g., earthworm) break down detritus into smaller particles. This process is called
fragmentation.
• By the process of leaching, water soluble inorganic nutrients go down into the soil horizon and get
precipitated as unavailable salts.
• Bacterial and fungal enzymes degrade detritus into simpler inorganic substances. This process is called as
catabolism.
• It is important to note that all the above steps in decomposition operate simultaneously on the detritus.
Humification and mineralisation occur during decomposition in the soil. Humification leads to accumulation
of a dark coloured amorphous substance called humus that is highly resistant to microbial action and
undergoes decomposition at an extremely slow rate. Being colloidal in nature it serves as a reservoir of
nutrients. The humus is further degraded by some microbes and release of inorganic nutrients occur by the
process known as mineralisation.

www.insightsonindia.com 52
INSTA STATIC QUIZ
12) Consider the following statements.
1. The standing crop is measured as the mass of living organisms (biomass) or the number in a unit area.
2. Plants capture only 30-40 per cent of the photosynthetically active radiation (PAR).
Which of the above statements is/are correct?
a) 1 only
b) 2 only
c) Both 1 and 2
d) Neither 1 nor 2

Solution: a)

Except for the deep-sea hydro-thermal ecosystem, sun is the only source of energy for all ecosystems on Earth. Of
the incident solar radiation less than 50 per cent of it is photosynthetically active radiation (PAR). We know that
plants and photosynthetic bacteria (autotrophs), fix Sun’s radiant energy to make food from simple inorganic
materials. Plants capture only 2-10 per cent of the PAR and this small amount of energy sustains the entire living
world.

Each trophic level has a certain mass of living material at a particular time called as the standing crop. The
standing crop is measured as the mass of living organisms (biomass) or the number in a unit area. The biomass of
a species is expressed in terms of fresh or dry weight. Measurement of biomass in terms of dry weight is more
accurate.

13) Which of the following ecosystem has an upright pyramid?


1. The pyramid of number in a tree ecosystem.
2. The pyramid of energy in a grassland ecosystem.
3. The pyramid of biomass in sea.
Select the correct answer code:
a) 1, 2
b) 2 only
c) 2, 3
d) 1, 3

Solution: b)

• In most ecosystems, all the pyramids, of number, of energy and biomass are upright, i.e., producers are more
in number and biomass than the herbivores, and herbivores are more in number and biomass than the
carnivores. Also, energy at a lower trophic level is always more than at a higher level.
• There are exceptions to this generalization: If you were to count the number of insects feeding on a big tree
what kind of pyramid would you get? Now add an estimate of the number of small birds depending on the
insects, as also the number of larger birds eating the smaller. Draw the shape you would get.
• The pyramid of biomass in sea is generally inverted because the biomass of fishes far exceeds that of
phytoplankton.
• Pyramid of energy is always upright, can never be inverted, because when energy flows from a particular
trophic level to the next trophic level, some energy is always lost as heat at each step. Each bar in the energy
pyramid indicates the amount of energy present at each trophic level in a given time or annually per unit
area.
• The number of trophic levels in the grazing food chain is restricted as the transfer of energy follows 10 per
cent law – only 10 per cent of the energy is transferred to each trophic level from the lower trophic level.

14) Consider the following statements.


1. In primary succession on rocks, pioneer species are usually lichens.
2. Hydrarch succession takes place in wet areas and the successional series progress from hydric to the
xeric conditions.
Which of the above statements is/are correct?
a) 1 only
www.insightsonindia.com 53
INSTA STATIC QUIZ
b) 2 only
c) Both 1 and 2
d) Neither 1 nor 2

Solution: a)

Succession of Plants:
Based on the nature of the habitat – whether it is water (or very wet areas) or it is on very dry areas – succession
of plants is called hydrarch or xerarch, respectively. Hydrarch succession takes place in wet areas and the
successional series progress from hydric to the mesic conditions.
As against this, xerarch succession takes place in dry areas and the series progress from xeric to mesic conditions.
Hence, both hydrarch and xerarch successions lead to medium water conditions (mesic) – neither too dry (xeric)
nor too wet (hydric).
The species that invade a bare area are called pioneer species. In primary succession on rocks these are usually
lichens which are able to secrete acids to dissolve rock, helping in weathering and soil formation. These later
pave way to some very small plants like bryophytes, which are able to take hold in the small amount of soil. They
are, with time, succeeded by higher plants, and after several more stages, ultimately a stable climax forest
community is formed. The climax community remains stable as long as the environment remains unchanged.
With time the xerophytic habitat gets converted into a mesophytic one.
Another important fact is to understand that all succession whether taking place in water or on land, proceeds to
a similar climax community – the mesic.

15) Which of the following are the reasons for greater biological diversity in tropics?
1. There is more solar energy available in the tropics, which contributes to higher productivity.
2. Tropical latitudes have remained relatively undisturbed for millions of years.
3. Tropical environments, unlike temperate ones, are less seasonal, relatively more constant and
predictable.
Select the correct answer code:
a) 1, 2
b) 1, 3
c) 2, 3
d) 1, 2, 3

Solution: d)

What is so special about tropics that might account for their greater biological diversity? Ecologists and
evolutionary biologists have proposed various hypotheses; some important ones are
(a) Speciation is generally a function of time, unlike temperate regions subjected to frequent glaciations in the
past, tropical latitudes have remained relatively undisturbed for millions of years and thus, had a long
evolutionary time for species diversification,
(b) Tropical environments, unlike temperate ones, are less seasonal, relatively more constant and predictable.
Such constant environments promote niche specialisation and lead to a greater species diversity and
(c) There is more solar energy available in the tropics, which contributes to higher productivity; this in turn
might contribute indirectly to greater diversity.

www.insightsonindia.com 54

You might also like